Presentation is loading. Please wait.

Presentation is loading. Please wait.

HIGHER – ADDITIONAL QUESTION BANK

Similar presentations


Presentation on theme: "HIGHER – ADDITIONAL QUESTION BANK"— Presentation transcript:

1 HIGHER – ADDITIONAL QUESTION BANK
Please decide which Unit you would like to revise: UNIT 1 UNIT 2 UNIT 3 Straight Line Functions & Graphs Trig Graphs & Equations Basic Differentiation Recurrence Relations Polynomials Quadratic Functions Integration Addition Formulae The Circle Vectors Further Calculus Exponential / Logarithmic Functions The Wave Function EXIT

2 HIGHER – ADDITIONAL QUESTION BANK
UNIT 1 : Straight Line Trig Graphs & Equations Functions & Graphs Basic Differentiation Recurrence Relations EXIT

3 HIGHER – ADDITIONAL QUESTION BANK
You have chosen to study: UNIT 1 : Straight Line Please choose a question to attempt from the following: 1 2 3 4 5 Back to Unit 1 Menu EXIT

4 STRAIGHT LINE : Question 1
Find the equation of the straight line which is perpendicular to the line with equation 3x – 5y = 4 and which passes through the point (-6,4). Reveal answer only Go to full solution Go to Marker’s Comments Go to Straight Line Menu Go to Main Menu EXIT

5 STRAIGHT LINE : Question 1
Find the equation of the straight line which is perpendicular to the line with equation 3x – 5y = 4 and which passes through the point (-6,4). y = -5/3x - 6 Reveal answer only Go to full solution Go to Marker’s Comments Go to Straight Line Menu Go to Main Menu EXIT

6 Question 1 (5) 3x – 5y = 4 Find the equation of the 3x - 4 = 5y
straight line which is perpendicular to the line with equation 3x – 5y = 4 and which passes through the point (-6,4). 3x - 4 = 5y 5y = 3x - 4 (5) y = 3/5x - 4/5 Using y = mx + c , gradient of line is 3/5 So required gradient = -5/3 , ( m1m2 = -1) We now have (a,b) = (-6,4) & m = -5/3 . Using y – b = m(x – a) We get y – 4 = -5/3 (x – (-6)) y – 4 = -5/3 (x + 6) Begin Solution y – 4 = -5/3x - 10 Continue Solution Markers Comments y = -5/3x - 6 Straight Line Menu Back to Home

7 (5) Markers Comments 3x – 5y = 4 3x - 4 = 5y 5y = 3x - 4
An attempt must be made to put the original equation into the form y = mx + c to read off the gradient. 3x – 5y = 4 3x - 4 = 5y 5y = 3x - 4 (5) State the gradient clearly. State the condition for perpendicular lines m1 m2 = -1. y = 3/5x - 4/5 Using y = mx + c , gradient of line is 3/5 When finding m2 simply invert and change the sign on m1 So required gradient = -5/3 , ( m1m2 = -1) m1 = 3 5 m2 = -5 3 We now have (a,b) = (-6,4) & m = -5/3 . Using y – b = m(x – a) Use the y - b = m(x - a) form to obtain the equation of the line. We get y – 4 = -5/3 (x – (-6)) y – 4 = -5/3 (x + 6) Next Comment y – 4 = -5/3x - 10 Straight Line Menu y = -5/3x - 6 Back to Home

8 STRAIGHT LINE : Question 2
Find the equation of the straight line which is parallel to the line with equation 8x + 4y – 7 = 0 and which passes through the point (5,-3). Reveal answer only Go to full solution Go to Marker’s Comments Go to Straight Line Menu Go to Main Menu EXIT

9 STRAIGHT LINE : Question 2
Find the equation of the straight line which is parallel to the line with equation 8x + 4y – 7 = 0 and which passes through the point (5,-3). y = -2x + 7 Reveal answer only Go to full solution Go to Marker’s Comments Go to Straight Line Menu Go to Main Menu EXIT

10 Question 2 (4) 8x + 4y – 7 = 0 Find the equation of the
straight line which is parallel to the line with equation 8x + 4y – 7 = 0 and which passes through the point (5,-3). 4y = -8x + 7 (4) y = -2x + 7/4 Using y = mx + c , gradient of line is -2 So required gradient = -2 as parallel lines have equal gradients. We now have (a,b) = (5,-3) & m = -2. Using y – b = m(x – a) We get y – (-3) = -2(x – 5) Begin Solution y + 3 = -2x + 10 Continue Solution Markers Comments y = -2x + 7 Straight Line Menu Back to Home

11 (4) Markers Comments An attempt must be made to
put the original equation into the form y = mx + c to read off the gradient. 8x + 4y – 7 = 0 4y = -8x + 7 (4) y = -2x + 7/4 State the gradient clearly. Using y = mx + c , gradient of line is -2 State the condition for parallel lines m1 = m2 So required gradient = -2 as parallel lines have equal gradients. Use the y - b = m(x - a) form to obtain the equation of the line. We now have (a,b) = (5,-3) & m = -2. Using y – b = m(x – a) We get y – (-3) = -2(x – 5) y + 3 = -2x + 10 Next Comment Straight Line Menu y = -2x + 7 Back to Home

12 STRAIGHT LINE : Question 3
X Y A B C In triangle ABC, A is (2,0), B is (8,0) and C is (7,3). (a) Find the gradients of AC and BC. (b) Hence find the size of ACB. Reveal answer only Go to full solution Go to Marker’s Comments Go to Straight Line Menu Go to Main Menu EXIT

13 STRAIGHT LINE : Question 3
X Y A B C In triangle ABC, A is (2,0), B is (8,0) and C is (7,3). (a) Find the gradients of AC and BC. (b) Hence find the size of ACB. mAC (a) = 3/5 Reveal answer only mBC = - 3 Go to full solution Go to Marker’s Comments (b) = 77.4° Go to Straight Line Menu Go to Main Menu EXIT

14 Question 3 Using the gradient formula:
In triangle ABC, A is (2,0), B is (8,0) and C is (7,3). mAC = 3 – = 3/5 Find the gradients of AC and BC. mBC = 3 – = - 3 (b) Hence find the size of ACB. (b) Using tan = gradient X Y A B C If tan = 3/5 then CAB = 31.0° If tan = -3 then CBX = ( )° = o so ABC = 71.6° Begin Solution Hence : ACB = 180° – 31.0° – 71.6° Continue Solution Markers Comments = 77.4° Straight Line Menu Back to Home

15 If no diagram is given draw a neat labelled diagram.
Markers Comments If no diagram is given draw a neat labelled diagram. In calculating gradients state the gradient formula. Using the gradient formula: mAC = 3 – = 3/5 Must use the result that the gradient of the line is equal to the tangent of the angle the line makes with the positive direction of the x-axis. Not given on the formula sheet. mBC = 3 – = - 3 (b) Using tan = gradient If tan = 3/5 then CAB = 31.0° A B Ø ° mAB = tanØ ° Ø ° = tan-1 mAB If tan = -3 then CBX = ( )° = o so ABC = 71.6° Hence : ACB = 180° – 31.0° – 71.6° Next Comment Straight Line Menu = 77.4° Back to Home

16 STRAIGHT LINE : Question 4
Y In triangle PQR the vertices are P(4,-5), Q(2,3) and R(10-1). Q(2,3) Find (a) the equation of the line e, the median from R of triangle PQR. X R(10,-1) P(4,-5) (b) the equation of the line f, the perpendicular bisector of QR. Reveal answer only (c) The coordinates of the point of intersection of lines e & f. Go to full solution Go to Marker’s Comments Go to Straight Line Menu Go to Main Menu EXIT

17 STRAIGHT LINE : Question 4
Y In triangle PQR the vertices are P(4,-5), Q(2,3) and R(10-1). Q(2,3) Find (a) the equation of the line e, the median from R of triangle PQR. X R(10,-1) P(4,-5) (b) the equation of the line f, the perpendicular bisector of QR. Reveal answer only (c) The coordinates of the point of intersection of lines e & f. Go to full solution Go to Marker’s Comments (a) y = -1 Go to Straight Line Menu (b) y = 2x – 11 Go to Main Menu EXIT (5,-1) (c)

18 Question 4 (a) mSR = -1 – (-1) 10 - 3
Midpoint of PQ is (3,-1): let’s call this S In triangle PQR the vertices are P(4,-5), Q(2,3) and R(10-1). Using the gradient formula m = y2 – y x2 – x1 Find (a) the equation of the line e, the median from R of triangle PQR. mSR = -1 – (-1) = 0 (ie line is horizontal) X Y P(4,-5) Q(2,3) R(10,-1) Since it passes through (3,-1) equation of e is y = -1 Solution to 4 (b) Begin Solution Continue Solution Markers Comments Straight Line Menu Back to Home

19 Question 4 (b) = 4/-8 = - 1/2 (b) Midpoint of QR is (6,1)
mQR = 3 – (-1) = 4/-8 In triangle PQR the vertices are P(4,-5), Q(2,3) and R(10-1). = - 1/2 Find required gradient = (m1m2 = -1) (b) the equation of the line f, the perpendicular bisector of QR. Using y – b = m(x – a) with (a,b) = (6,1) & m = 2 X Y P(4,-5) Q(2,3) R(10,-1) we get y – 1 = 2(x – 6) so f is y = 2x – 11 Solution to 4 (c) Begin Solution Continue Solution Markers Comments Straight Line Menu Back to Home

20 Question 4 (c) (c) e & f meet when y = -1 & y = 2x -11
In triangle PQR the vertices are P(4,-5), Q(2,3) and R(10-1). so 2x – 11 = -1 Find ie 2x = 10 (c) The coordinates of the point of intersection of lines e & f. ie x = 5 X Y P(4,-5) Q(2,3) R(10,-1) Point of intersection is (5,-1) Begin Solution Continue Solution Markers Comments Straight Line Menu Back to Home

21 If no diagram is given draw a neat labelled diagram.
Markers Comments If no diagram is given draw a neat labelled diagram. Midpoint of PQ is (3,-1): let’s call this S Using the gradient formula m = y2 – y x2 – x1 Sketch the median and the perpendicular bisector mSR = -1 – (-1) (ie line is horizontal) median Q P R y x Perpendicular bisector Since it passes through (3,-1) equation of e is y = -1 Comments for 4 (b) Next Comment Straight Line Menu Back to Home

22 , = 4/-8 = - 1/2 Q R P Markers Comments To find midpoint of QR
(b) Midpoint of QR is (6,1) mQR = 3 – (-1) = 4/-8 (-1) , Look for special cases: = - 1/2 Horizontal lines in the form y = k Vertical lines in the form x = k required gradient = (m1m2 = -1) Using y – b = m(x – a) with (a,b) = (6,1) & m = 2 we get y – 1 = 2(x – 6) Q P R y x so f is y = 2x – 11 Next Comment Comments for 4 (c) Straight Line Menu Back to Home

23 Markers Comments To find the point of intersection of the two lines solve the two equations: (c) e & f meet when y = -1 & y = 2x -11 so 2x – 11 = -1 ie 2x = 10 ie x = 5 y = -1 y = 2x - 11 Point of intersection is (5,-1) Next Comment Straight Line Menu Back to Home

24 STRAIGHT LINE : Question 5
X Y G(2,-5) E(6,-3) F(12,-5) In triangle EFG the vertices are E(6,-3), F(12,-5) and G(2,-5). Find (a) the equation of the altitude from vertex E. (b) the equation of the median from vertex F. (c) The point of intersection of the altitude and median. Reveal answer only Go to full solution Go to Marker’s Comments Go to Straight Line Menu Go to Main Menu EXIT

25 STRAIGHT LINE : Question 5
X Y G(2,-5) E(6,-3) F(12,-5) In triangle EFG the vertices are E(6,-3), F(12,-5) and G(2,-5). Find (a) the equation of the altitude from vertex E. (b) the equation of the median from vertex F. (c) The point of intersection of the altitude and median. Reveal answer only Go to full solution (a) x = 6 Go to Marker’s Comments (b) x + 8y + 28 = 0 Go to Straight Line Menu (c) Go to Main Menu (6,-4.25) EXIT

26 Question 5(a) mFG = -5 – (-5) 12 - 2 Using the gradient formula
In triangle EFG the vertices are E(6,-3), F(12,-5) and G(2,-5). mFG = -5 – (-5) = 0 Find (a) the equation of the altitude from vertex E. (ie line is horizontal so altitude is vertical) Y Altitude is vertical line through (6,-3) ie x = 6 X E(6,-3) F(12,-5) G(2,-5) Solution to 5 (b) Begin Solution Continue Solution Markers Comments Straight Line Menu Back to Home

27 Question 5(b) = -1/8 (X8) Midpoint of EG is (4,-4)- let’s call this H
In triangle EFG the vertices are E(6,-3), F(12,-5) and G(2,-5). mFH = -5 – (-4) = -1/8 Find Using y – b = m(x – a) with (a,b) = (4,-4) & m = -1/8 (b) the equation of the median from vertex F. Y we get y – (-4) = -1/8(x – 4) (X8) X E(6,-3) or y + 32 = -x + 4 F(12,-5) G(2,-5) Median is x + 8y + 28 = 0 Begin Solution Solution to 5 (c) Continue Solution Markers Comments Straight Line Menu Back to Home

28 Question 5(c) (c) Lines meet when x = 6 & x + 8y + 28 = 0
In triangle EFG the vertices are E(6,-3), F(12,-5) and G(2,-5). put x =6 in 2nd equation y + 34 = 0 Find ie y = -34 (c) The point of intersection of the altitude and median. ie y = -4.25 Y Point of intersection is (6,-4.25) X E(6,-3) F(12,-5) G(2,-5) Begin Solution Continue Solution Markers Comments Straight Line Menu Back to Home

29 mFG = -5 – (-5) 12 - 2 E F G Markers Comments
If no diagram is given draw a neat labelled diagram. Sketch the altitude and the median. Using the gradient formula mFG = -5 – (-5) = 0 y x F E G (ie line is horizontal so altitude is vertical) Altitude is vertical line through (6,-3) ie x = 6 median Comments for 5 (b) altitude Next Comment Straight Line Menu Back to Home

30 = -1/8 (X8) E F G Markers Comments To find midpoint of EG
(-5) , H Midpoint of EG is (4,-4)- call this H Look for special cases: mFH = -5 – (-4) = -1/8 Horizontal lines in the form y = k Vertical lines in the form x = k Using y – b = m(x – a) with (a,b) = (4,-4) & m = -1/8 y x F E G we get y – (-4) = -1/8(x – 4) (X8) or y + 32 = -x + 4 Median is x + 8y + 28 = 0 Next Comment Straight Line Menu Comments for 5 (c) Back to Home

31 intersection of the two lines solve the two equations: (c)
Markers Comments To find the point of intersection of the two lines solve the two equations: (c) Lines meet when x = 6 & x + 8y + 28 = 0 x = 6 x + 8y = -28 put x =6 in 2nd equation y + 34 = 0 ie y = -34 ie y = -4.25 Point of intersection is (6,-4.25) Next Comment Straight Line Menu Back to Home

32 Basic Differentiation
HIGHER – ADDITIONAL QUESTION BANK You have chosen to study: UNIT 1 : Basic Differentiation Please choose a question to attempt from the following: 1 2 3 4 5 Back to Unit 1 Menu EXIT

33 BASIC DIFFERENTIATION : Question 1
Find the equation of the tangent to the curve (x>0) at the point where x = 4. Reveal answer only Go to full solution Go to Marker’s Comments Go to Basic Differentiation Menu Go to Main Menu EXIT

34 BASIC DIFFERENTIATION : Question 1
Find the equation of the tangent to the curve (x>0) at the point where x = 4. Reveal answer only y = 5/4x – 7 Go to full solution Go to Marker’s Comments Go to Basic Differentiation Menu Go to Main Menu EXIT

35 Question 1 NB: a tangent is a line so we need a point of contact and a gradient. Find the equation of the tangent to the curve y = x – 16 x (x>0) at the point where x = 4. Point If x = 4 then y = 4 – = 2 – 4 = -2 so (a,b) = (4,-2) Gradient: y = x – x = x1/2 – 16x -1 Continue Solution dy/dx = 1/2x-1/2 + 16x-2 = x x2 If x = 4 then: Begin Solution dy/dx =  Continue Solution = ¼ = 5/4 Markers Comments Basic Differentiation Menu Back to Home

36 Question 1 If x = 4 then: dy/dx = 1 + 16 24 16 = ¼ + 1 = 5/4
Find the equation of the tangent to the curve y = x – 16 x (x>0) at the point where x = 4. dy/dx =  = ¼ = 5/4 Gradient of tangent = gradient of curve so m = 5/4 . We now use y – b = m(x – a) this gives us y – (-2) = 5/4(x – 4) Back to Previous or y = 5/4x – 5 Begin Solution or y = 5/4x – 7 Continue Solution Markers Comments Basic Differentiation Menu Back to Home

37 Prepare expression for differentiation.
Markers Comments Prepare expression for differentiation. NB: a tangent is a line so we need a point of contact and a gradient. Point Find gradient of the tangent using rule: If x = 4 then y = 4 – = 2 – 4 = -2 “multiply by the power and reduce the power by 1” so (a,b) = (4,-2) Gradient: y = x – x = x1/2 – 16x -1 Find gradient = at x = 4. dy/dx = 1/2x-1/2 + 16x-2 = x x2 Continue Comments If x = 4 then: dy/dx =  Next Comment Differentiation Menu = ¼ = 5/4 Back to Home

38 Find y coordinate at x = 4 using: If x = 4 then:
Markers Comments Find y coordinate at x = 4 using: If x = 4 then: dy/dx =  = ¼ = 5/4 Use m = 5/4 and (4,-2) in y - b = m(x - a) Gradient of tangent = gradient of curve so m = 5/4 . We now use y – b = m(x – a) this gives us y – (-2) = 5/4(x – 4) or y = 5/4x – 5 or y = 5/4x – 7 Next Comment Differentiation Menu Back to Home

39 BASIC DIFFERENTIATION : Question 2
Find the coordinates of the point on the curve y = x2 – 5x where the tangent to the curve makes an angle of 135° with the positive direction of the X-axis. Reveal answer only Go to full solution Go to Marker’s Comments Go to Basic Differentiation Menu Go to Main Menu EXIT

40 BASIC DIFFERENTIATION : Question 2
Find the coordinates of the point on the curve y = x2 – 5x where the tangent to the curve makes an angle of 135° with the positive direction of the X-axis. (2,4) Reveal answer only Go to full solution Go to Marker’s Comments Go to Basic Differentiation Menu Go to Main Menu EXIT

41 Question 2 NB: gradient of line = gradient of curve
Find the coordinates of the point on the curve y = x2 – 5x + 10 where the tangent to the curve makes an angle of 135° with the positive direction of the X-axis. Line Using gradient = tan we get gradient of line = tan135° = -tan45° = -1 Curve Gradient of curve = dy/dx = 2x - 5 Continue Solution It now follows that 2x – 5 = -1 Begin Solution Or x = 4 Continue Solution Or x = 2 Markers Comments Basic Differentiation Menu Back to Home

42 Question 2 Using y = x2 – 5x + 10 with x = 2
Find the coordinates of the point on the curve y = x2 – 5x + 10 where the tangent to the curve makes an angle of 135° with the positive direction of the X-axis. we get y = 22 – (5 X 2) + 10 ie y = 4 So required point is (2,4) Back to Previous Begin Solution Continue Solution Markers Comments Basic Differentiation Menu Back to Home

43 m = tan135 ° = -1 Markers Comments Find gradient of the tangent
using rule: NB: gradient of line = gradient of curve “multiply by the power and reduce the power by 1” Line Using gradient = tan Must use the result that the gradient of the line is also equal to the tangent of the angle the line makes with the positive direction of the x- axis. Not given on the formula sheet. we get gradient of line = tan135° = -tan45° = -1 Curve y x 135 ° Gradient of curve = dy/dx = 2x - 5 m = tan135 ° = -1 It now follows that 2x – 5 = -1 Or x = 4 Next Comment Or x = 2 Differentiation Menu Continue Comments Back to Home

44 Set m = i.e. 2x - 5 = -1 and solve for x. Markers Comments
It now follows that 2x – 5 = -1 Or x = 4 Or x = 2 Using y = x2 – 5x + 10 with x = 2 we get y = 22 – (5 X 2) + 10 ie y = 4 So required point is (2,4) Next Comment Differentiation Menu Back to Home

45 BASIC DIFFERENTIATION : Question 3
y = g(x) The graph of y = g(x) is shown here. There is a point of inflection at the origin, a minimum turning point at (p,q) and the graph also cuts the X-axis at r. r Make a sketch of the graph of y = g(x). (p,q) Reveal answer only Go to full solution Go to Marker’s Comments Go to Basic Differentiation Menu EXIT Go to Main Menu

46 BASIC DIFFERENTIATION : Question 3
y = g(x) The graph of y = g(x) is shown here. There is a point of inflection at the origin, a minimum turning point at (p,q) and the graph also cuts the X-axis at r. r Make a sketch of the graph of y = g(x). (p,q) Reveal answer only y = g(x) Go to full solution Go to Marker’s Comments Go to Basic Differentiation Menu Go to Main Menu EXIT

47 Question 3 Stationary points occur at x = 0 and x = p.
y = g(x) Stationary points occur at x = 0 and x = p. (We can ignore r.) We now consider the sign of the gradient either side of 0 and p: r x   p  g(x) (p,q) Make a sketch of the graph of y = g(x). new y-values Begin Solution Click for graph Continue Solution Markers Comments Basic Differentiation Menu Back to Home

48 Question 3 This now gives us the following graph
y = g(x) This now gives us the following graph y = g(x) r p (p,q) Make a sketch of the graph of y = g(x). Begin Solution Return to Nature Table Continue Solution Markers Comments Basic Differentiation Menu Back to Home

49 the gradient function: Stationary points occur at x = 0 and x = p.
Markers Comments To sketch the graph of the gradient function: Stationary points occur at x = 0 and x = p. 1 Mark the stationary points on the x axis i.e. p y = g(x) x y a Continue Comments Next Comment Differentiation Menu Back to Home

50 + - - Markers Comments To sketch the graph of the gradient function:
Stationary points occur at x = 0 and x = p. (We can ignore r.) 1 Mark the stationary points on the x axis i.e. We now consider the sign of the gradient either side of 0 and p: 2 For each interval decide if the value of x   p  g(x) y a + x new y-values - - Next Comment Differentiation Menu Continue Comments Back to Home

51 + - Markers Comments To sketch the graph of the gradient function:
Stationary points occur at x = 0 and x = p. 1 Mark the stationary points on the x axis i.e. p y = g(x) 2 For each interval decide if the value of 3 Draw in curve to fit information + - x y a In any curve sketching question use a ruler and annotate the sketch i.e. label all known coordinates. Next Comment Differentiation Menu Back to Home

52 BASIC DIFFERENTIATION : Question 4
Here is part of the graph of y = x3 - 3x2 - 9x + 2. y = x3 - 3x2 - 9x + 2 Find the coordinates of the stationary points and determine their nature algebraically. Reveal answer only Go to full solution Go to Marker’s Comments Go to Basic Differentiation Menu Go to Main Menu EXIT

53 BASIC DIFFERENTIATION : Question 4
Here is part of the graph of y = x3 - 3x2 - 9x + 2. y = x3 - 3x2 - 9x + 2 Find the coordinates of the stationary points and determine their nature algebraically. Reveal answer only (-1,7) is a maximum TP and (3,-25) is a minimum TP Go to full solution Go to Marker’s Comments Go to Basic Differentiation Menu Go to Main Menu EXIT

54 BASIC DIFFERENTIATION : Question 4
Here is part of the graph of y = x3 - 3x2 - 9x + 2. y = x3 - 3x2 - 9x + 2 Find the coordinates of the stationary points and determine their nature algebraically. Return to solution EXIT

55 Question 4 SPs occur where dy/dx = 0 ie 3x2 – 6x – 9 = 0
Here is part of the graph of y = x3 - 3x2 - 9x + 2. ie (x2 – 2x – 3) = 0 ie (x – 3)(x + 1) = 0 Find the coordinates of the stationary points and determine their nature algebraically. ie x = -1 or x = 3 Using y = x3 - 3x2 - 9x + 2 when x = -1 Continue Solution y = -1 – = 7 & when x = 3 y = 27 – = -25 Begin Solution So stationary points are at (-1,7) and (3,-25) Continue Solution Markers Comments Basic Differentiation Menu Back to Home

56 Question 4 We now consider the sign of the gradient
either side of -1 and 3. Here is part of the graph of y = x3 - 3x2 - 9x + 2. x   3  Find the coordinates of the stationary points and determine their nature algebraically. (x + 1) (x - 3) dy/dx Back to graph Begin Solution Hence (-1,7) is a maximum TP and (3,-25) is a minimum TP Continue Solution Markers Comments Basic Differentiation Menu Back to Home

57 “At stationary points “ SPs occur where dy/dx = 0
Markers Comments Make the statement: “At stationary points “ SPs occur where dy/dx = 0 ie 3x2 – 6x – 9 = 0 ie (x2 – 2x – 3) = 0 Must attempt to find and set equal to zero ie (x – 3)(x + 1) = 0 ie x = -1 or x = 3 “multiply by the power and reduce the power by 1” Using y = x3 - 3x2 - 9x + 2 when x = -1 Find the value of y from y = x3 -3x2-9x+2 not from y = -1 – = 7 & when x = 3 y = 27 – = -25 So stationary points are at (-1,7) and (3,-25) Next Comment Differentiation Menu Continue Comments Back to Home

58 x -1 + 0 - Minimum requirement Markers Comments
Justify the nature of each stationary point using a table of “signs” We now consider the sign of the gradient either side of -1 and 3. x -1 x   3  (x + 1) Minimum requirement (x - 3) dy/dx State the nature of the stationary point i.e. Maximum T.P. Hence (-1,7) is a maximum TP and (3,-25) is a minimum TP Next Comment Differentiation Menu Back to Home

59 BASIC DIFFERENTIATION : Question 5
When a company launches a new product its share of the market after x months is calculated by the formula S(x) = x x2 (x  2) So after 5 months the share is S(5) = 2/5 – 4/25 = 6/25 Find the maximum share of the market that the company can achieve. Reveal answer only Go to full solution Go to Marker’s Comments Go to Basic Differentiation Menu EXIT Go to Main Menu

60 BASIC DIFFERENTIATION : Question 5
When a company launches a new product its share of the market after x months is calculated by the formula S(x) = x x2 (x  2) So after 5 months the share is S(5) = 2/5 – 4/25 = 6/25 Find the maximum share of the market that the company can achieve. Reveal answer only = 1/4 Go to full solution Go to Marker’s Comments Go to Basic Differentiation Menu EXIT Go to Main Menu

61 = 8 - 2 x3 x2 Question 5 End points When a company launches a new
product its share of the market after x months is calculated as: S(2) = 1 – 1 = 0 There is no upper limit but as x   S(x)  0. S(x) = x x2 (x  2) Stationary Points S(x) = x x2 = 2x-1 – 4x-2 Find the maximum share of the market that the company can achieve. So S (x) = -2x-2 + 8x-3 = x3 x2 = x2 x3 Begin Solution Continue Solution Continue Solution Markers Comments Basic Differentiation Menu Back to Home

62 Question 5 SPs occur where S (x) = 0 When a company launches a new
product its share of the market after x months is calculated as: x3 x2 = 0 8 = x3 x2 ( cross mult!) or S(x) = x x2 (x  2) 8x2 = 2x3 8x2 - 2x3 = 0 2x2(4 – x) = 0 Find the maximum share of the market that the company can achieve. x = 0 or x = 4 In required interval NB: x  2 Go Back to Previous Continue Solution Continue Solution Markers Comments Basic Differentiation Menu Back to Home

63 Question 5 We now check the gradients either side of X = 4
When a company launches a new product its share of the market after x months is calculated as: x   S (3.9 ) = … S(x) = x x2 (x  2) S (x) S (4.1) = … Find the maximum share of the market that the company can achieve. Hence max TP at x = 4 So max share of market = S(4) = 2/4 – 4/16 Go Back to Previous = 1/2 – 1/4 Continue Solution = 1/4 Markers Comments Basic Differentiation Menu Back to Home

64 = 8 - 2 x3 x2 Markers Comments Must look for key word to spot
the optimisation question i.e. End points Maximum, minimum, greatest , least etc. S(2) = 1 – 1 = 0 There is no upper limit but as x   S(x)  0. Must consider end points and stationary points. Stationary Points S(x) = x x2 = 2x-1 – 4x-2 Prepare expression for differentiation. So S (x) = -2x-2 + 8x-3 = x3 x2 = x2 x3 Next Comment Differentiation Menu Continue Comments Back to Home

65 ( Note: No marks are allocated for trial and error solution.)
Markers Comments Must attempt to find ( Note: No marks are allocated for trial and error solution.) SPs occur where S (x) = 0 x3 x2 = 0 Must attempt to find and set equal to zero 8 = x3 x2 ( cross mult!) or 8x2 = 2x3 “multiply by the power and reduce the power by 1” 8x2 - 2x3 = 0 Usually easier to solve resulting equation using cross-multiplication. Take care to reject “solutions” outwith the domain. 2x2(4 – x) = 0 x = 0 or x = 4 In required interval NB: x  2 Next Comment Differentiation Menu Continue Comments Back to Home

66 x 4 + 0 - Minimum requirement. Markers Comments
Must show a maximum value using a table of “signs”. We now check the gradients either side of X = 4 x 4 + 0 - x   S (3.9 ) = … S (x) S (4.1) = … Minimum requirement. Hence max TP at x = 4 State clearly: Maximum T.P at x = 4 So max share of market = S(4) = 2/4 – 4/16 Next Comment = 1/2 – 1/4 Differentiation Menu = 1/4 Back to Home

67 HIGHER – ADDITIONAL QUESTION BANK
You have chosen to study: UNIT 1 : Recurrence Relations Please choose a question to attempt from the following: 1 2 3 Back to Unit 1 Menu EXIT

68 RECURRENCE RELATIONS : Question 1
A recurrence relation is defined by the formula un+1 = aun + b, where -1<a<1 and u0 = 20. If u1 = 10 and u2 = 4 then find the values of a and b. Find the limit of this recurrence relation as n  . Reveal answer only Go to full solution Go to Marker’s Comments Go to Basic Differentiation Menu EXIT Go to Main Menu

69 RECURRENCE RELATIONS : Question 1
A recurrence relation is defined by the formula un+1 = aun + b, where -1<a<1 and u0 = 20. If u1 = 10 and u2 = 4 then find the values of a and b. Find the limit of this recurrence relation as n  . (a) a = 0.6 Reveal answer only b = -2 Go to full solution (b) Go to Marker’s Comments L = -5 Go to Basic Differentiation Menu EXIT Go to Main Menu

70 Question 1 (a) Using un+1 = aun + b A recurrence relation is defined
by the formula un+1 = aun + b, where -1<a<1 and u0 = 20. If u1 = 10 and u2 = 4 then find the values of a and b. (b) Find the limit of this recurrence relation as n  . we get u1 = au0 + b and u2 = au1 + b Replacing u0 by 20, u1 by 10 & u2 by 4 gives us 20a + b = 10 and 10a + b = 4 subtract  a = 6 or a = 0.6 Continue Solution Replacing a by 0.6 in 10a + b = 4 Begin Solution gives b = 4 Continue Solution or b = -2 Markers Comments Recurrence Relations Menu Back to Home

71 Question 1 (b) un+1 = aun + b is now un+1 = 0.6un - 2
A recurrence relation is defined by the formula un+1 = aun + b, where -1<a<1 and u0 = 20. If u1 = 10 and u2 = 4 then find the values of a and b. (b) Find the limit of this recurrence relation as n  . This has a limit since -1<0.6<1 At this limit, L, un+1 = un = L So we now have L = 0.6 L - 2 or L = -2 or L = -2  0.4 or L = -20  4 so L = -5 Begin Solution Continue Solution Markers Comments Recurrence Relations Menu Back to Home

72 simultaneous equations and solve. u1 is obtained from u0 and
Markers Comments Must form the two simultaneous equations and solve. u1 is obtained from u0 and u2 is obtained from u1 . A trial and error solution would only score 1 mark. (a) Using un+1 = aun + b we get u1 = au0 + b and u2 = au1 + b Replacing u0 by 20, u1 by 10 & u2 by 4 gives us 20a + b = 10 and 10a + b = 4 subtract  a = 6 Comments for 1(b) or a = 0.6 Replacing a by 0.6 in 10a + b = 4 gives b = 4 Next Comment Recurrence Menu or b = -2 Back to Home

73 Must state condition for limit i.e. -1 < 0.6 < 1
Markers Comments Must state condition for limit i.e. -1 < 0.6 < 1 At limit L, state un+1 = un = L Substitute L for un+1 and un and solve for L. (b) un+1 = aun + b is now un+1 = 0.6un - 2 This has a limit since -1<0.6<1 At this limit, L, un+1 = un = L So we now have L = 0.6 L - 2 or L = -2 or L = -2  0.4 or L = -20  4 so L = -5 Next Comment Recurrence Menu Back to Home

74 RECURRENCE RELATIONS : Question 2
Two different recurrence relations are known to have the same limit as n   The first is defined by the formula un+1 = -5kun The second is defined by the formula vn+1 = k2vn Find the value of k and hence this limit. Reveal answer only Go to full solution Go to Marker’s Comments Go to Recurrence Relations Menu EXIT Go to Main Menu

75 RECURRENCE RELATIONS : Question 2
Two different recurrence relations are known to have the same limit as n   The first is defined by the formula un+1 = -5kun The second is defined by the formula vn+1 = k2vn Find the value of k and hence this limit. k = 1/3 Reveal answer only L = 9/8 Go to full solution Go to Marker’s Comments Go to Basic Differentiation Menu EXIT Go to Main Menu

76 Question 2 If the limit is L then as n   we have
un+1 = un = L and vn+1 = vn = L Two different recurrence relations are known to have the same limit as n   The first is defined by the formula un+1 = -5kun + 3. The second is defined by Vn+1 = k2vn + 1. Find the value of k and hence this limit. First Sequence Second Sequence un+1 = -5kun + 3 becomes vn+1 = k2vn + 1 becomes L = k2L + 1 L = -5kL + 3 L - k2L = 1 L + 5kL = 3 L(1 - k2) = 1 L(1 + 5k) = 3 L = (1 - k2) L = (1 + 5k) Begin Solution Continue Solution Markers Comments Continue Solution Recurrence Relations Menu Back to Home

77 Question 2 L = 3 . . (1 + 5k) L = 1 . . (1 - k2)
Two different recurrence relations are known to have the same limit as n   The first is defined by the formula un+1 = -5kun + 3. The second is defined by Vn+1 = k2vn + 1. Find the value of k and hence this limit. = (1 + 5k) (1 – k2) It follows that Cross multiply to get k = 3 – 3k2 This becomes k2 + 5k – 2 = 0 Or (3k – 1)(k + 2) = 0 So k = 1/3 or k = -2 Since -1<k<1 then k = 1/3 Begin Solution Continue Solution Markers Comments Continue Solution Recurrence Relations Menu Back to Home

78 Question 1 Since -1<k<1 then k = 1/3
Two different recurrence relations are known to have the same limit as n   The first is defined by the formula un+1 = -5kun + 3. The second is defined by Vn+1 = k2vn + 1. Find the value of k and hence this limit. Using L = (1 - k2) gives us L = (1 – 1/9) or L = 1  8/9 ie L = 9/8 Begin Solution Continue Solution Markers Comments Recurrence Relations Menu Back to Home

79 Since both recurrence relations have the same limit, L, find the
Markers Comments Since both recurrence relations have the same limit, L, find the limit for both and set equal. If the limit is L then as n   we have un+1 = un = L and vn+1 = vn = L First Sequence Second Sequence un+1 = -5kun + 3 becomes vn+1 = k2vn + 1 becomes L = k2L + 1 L = -5kL + 3 L - k2L = 1 L + 5kL = 3 Continue Comments L(1 - k2) = 1 L(1 + 5k) = 3 L = (1 + 5k) L = (1 - k2) Next Comment Recurrence Menu Back to Home

80 Since both recurrence relations have the same limit, L, find the
Markers Comments Since both recurrence relations have the same limit, L, find the limit for both and set equal. L = (1 + 5k) L = (1 - k2) Only one way to solve resulting equation i.e. terms to the left, form the quadratic and factorise. State clearly the condition for the recurrence relation to approach a limit. -1< k < 1. Take care to reject the “solution” which is outwith the range. = (1 + 5k) (1 – k2) It follows that Cross multiply to get k = 3 – 3k2 This becomes k2 + 5k – 2 = 0 Or (3k – 1)(k + 2) = 0 So k = 1/3 or k = -2 Since -1<k<1 then k = 1/3 Next Comment Continue Solution Recurrence Menu Back to Home

81 Find L from either formula. Since -1<k<1 then k = 1/3
Markers Comments Find L from either formula. Since -1<k<1 then k = 1/3 Using L = (1 - k2) gives us L = (1 – 1/9) or L = 1  8/9 ie L = 9/8 Next Comment Recurrence Menu Back to Home

82 RECURRENCE RELATIONS : Question 3
A man plants a row of fast growing trees between his own house and his neighbour’s. These trees are known grow at a rate of 1m per annum so cannot be allowed to become too high. He therefore decides to prune 30% from their height at the beginning of each year. Using the 30% pruning scheme what height should he expect the trees to grow to in the long run? The neighbour is concerned at the growth rate and asks that the trees be kept to a maximum height of 3m. What percentage should be pruned to ensure that this happens? Reveal answer only Go to full solution Go to Marker’s Comments Go to Recurrence Relations Menu EXIT

83 RECURRENCE RELATIONS : Question 3
A man plants a row of fast growing trees between his own house and his neighbour’s. These trees are known grow at a rate of 1m per annum so cannot be allowed to become too high. He therefore decides to prune 30% from their height at the beginning of each year. Using the 30% pruning scheme what height should he expect the trees to grow to in the long run? The neighbour is concerned at the growth rate and asks that the trees be kept to a maximum height of 3m. What percentage should be pruned to ensure that this happens? Reveal answer only Height of trees in long run is 31/3m. Go to full solution (b) 331/3% Go to Marker’s Comments Go to Recurrence Relations Menu EXIT

84 Question 3 (a) Removing 30% leaves 70% or 0.7
The trees are known grow at a rate of 1m per annum. He therefore decides to prune 30% from their height at the beginning of each year. If Hn is the tree height in year n then Hn+1 = 0.7Hn + 1 Since -1<0.7<1 this sequence has a limit, L. At the limit Hn+1 = Hn = L Using the 30% pruning scheme what height should he expect the trees to grow to in the long run? So L= 0.7L + 1 or L = 1 ie L = 1  0.3 = 10  3 = 31/3 Height of trees in long run is 31/3m. Begin Solution Continue Solution Continue Solution Markers Comments Recurrence Relation Menu Back to Home

85 Question 3 If fraction left after pruning is a and
we need the limit to be 3 The trees are known grow at a rate of 1m per annum. He therefore decides to prune 30% from their height at the beginning of each year. then we have 3 = a X 3 + 1 or 3a = 2 or a = 2/3 (b) The neighbour asks that the trees be kept to a maximum height of 3m. What percentage should be pruned to ensure that this happens? This means that the fraction pruned is 1/3 or 331/3% Begin Solution Continue Solution Markers Comments Recurrence Relation Menu Back to Home

86 Do some numerical work to get the “feel” for the problem.
Markers Comments Do some numerical work to get the “feel” for the problem. (a) Removing 30% leaves 70% or 0.7 H0 = 1 (any value acceptable) H1 = 0.7 x = 1.7 H2 = 0.7 x = etc. If Hn is the tree height in year n then Hn+1 = 0.7Hn + 1 State the recurrence relation, with the starting value. Hn+1 = 0.7 Hn + 1, H0 = 1 State the condition for the limit -1< 0.7< 1 At limit L, state Hn+1 = Hn = L Substitute L for Hn+1 and Hn and solve for L. Since -1<0.7<1 this sequence has a limit, L. At the limit Hn+1 = Hn = L So L= 0.7L + 1 or L = 1 ie L = 1  0.3 = 10  3 Height of trees in long run is 31/3m. Next Comment Continue Solution Recurrence Menu Back to Home

87 Since we know the limit we are working backwards to %. L = 0.7L + 1
Markers Comments Since we know the limit we are working backwards to %. L = 0.7L + 1 New limit, L = 3 and multiplier a 3 = a x3 + 1 etc. Take care to subtract from 1 to get fraction pruned. If fraction left after pruning is a and we need the limit to be 3 then we have 3 = a X 3 + 1 or 3a = 2 or a = 2/3 This means that the fraction pruned is 1/3 or 331/3% Next Comment Recurrence Menu Back to Home

88 Trig Graphs & Equations
HIGHER – ADDITIONAL QUESTION BANK You have chosen to study: UNIT 1 : Trig Graphs & Equations Please choose a question to attempt from the following: 1 2 3 Back to Unit 1 Menu EXIT

89 TRIG GRAPHS & EQUATIONS : Question 1
/2 y = acosbx + c This diagram shows the graph of y = acosbx + c. Determine the values of a, b & c. Reveal answer only Go to full solution Go to Marker’s Comments Go to Trig Graphs & Equations Menu Go to Main Menu EXIT

90 TRIG GRAPHS & EQUATIONS : Question 1
/2 y = acosbx + c This diagram shows the graph of y = acosbx + c. Determine the values of a, b & c. Reveal answer only a = 3 Go to full solution b = 2 Go to Marker’s Comments c = -1 Go to Trig Graphs & Equations Menu Go to Main Menu EXIT

91 Question 1 a = ½(max – min)
This diagram shows the graph of y = acosbx + c. Determine the values of a, b & c. = ½(2 – (-4)) = ½ X 6 = 3 /2 y = acosbx + c Period of graph =  so two complete sections between 0 & 2 ie b = 2 For 3cos(…) max = 3 & min = -3. This graph: max = 2 & min = -4. ie 1 less Begin Solution so c = -1 Continue Solution Markers Comments Trig Graphs etc. Menu Back to Home

92 The values chosen for a,b and c must be justified.
Markers Comments The values chosen for a,b and c must be justified. Possible justification of a = 3 a = 1/2(max - min) y = cosx graph stretched by a factor of 3 etc. Possible justification of b = 2 Period of graph = 2 complete cycles in 2 ÷ = complete cycles in etc. Possible justification for c = -1 3cos max = 3, min = -3 This graph: max = 2, min = -4 i.e c = -1 y = 3cosx graph slide down 1 unit etc. a = ½(max – min) = ½(2 – (-4)) = ½ X 6 = 3 Period of graph =  so two complete sections between 0 & 2 ie b = 2 For 3cos(…) max = 3 & min = -3. This graph: max = 2 & min = -4. ie 1 less so c = -1 Next Comment Trig Graphs Menu Back to Home

93 TRIG GRAPHS & EQUATIONS : Question 2
Solve 3tan2 + 1 = 0 ( where 0 <  <  ). Reveal answer only Go to full solution Go to Marker’s Comments Go to Trig Graphs & Equations Menu Go to Main Menu EXIT

94 TRIG GRAPHS & EQUATIONS : Question 2
Solve 3tan2 + 1 = 0 ( where 0 <  <  ).  = 5/12 Reveal answer only  = 11/12 Go to full solution Go to Marker’s Comments Go to Trig Graphs & Equations Menu Go to Main Menu EXIT

95 Question 2 3tan2 + 1 = 0 3tan2 = -1 Solve 3tan2 + 1 = 0
( where 0 <  <  ). tan2 = -1/3 Q2 or Q4  -    +  2 -  sin all tan cos tan -1(1/3) = /6 Q2: angle =  - /6 so 2 = 5/6 ie  = 5/12 1 2 3 /6 Q4: angle = 2 - /6 so 2 = 11/6 Begin Solution ie  = 11/12 Continue Solution Markers Comments tan2 repeats every /2 radians but repeat values are not in interval. Trig Graphs etc. Menu Back to Home

96 Solve the equation for tan . Full marks can be obtained by
Markers Comments Solve the equation for tan Full marks can be obtained by working in degrees and changing final answers back to radians. 3tan2 + 1 = 0 Use the positive value when finding tan-1. 3tan2 = -1 tan2 = -1/3 Q2 or Q4 Use the quadrant rule to find the solutions.  -    +  2 -  sin all tan cos tan -1(1/3) = /6 Must learn special angles or be able to calculate from triangles. Q2: angle =  - /6 so 2 = 5/6 1 45° 2 60° 30° Take care to reject ”solutions” outwith domain. ie  = 5/12 Q4: angle = 2 - /6 1 2 3 /6 so 2 = 11/6 ie  = 11/12 Next Comment Trig Graphs Menu tan2 repeats every /2 radians but repeat values are not in interval. Back to Home

97 TRIG GRAPHS & EQUATIONS : Question 3
2/3 P Q y = 2 The diagram shows a the graph of a sine function from 0 to 2/3. (a) State the equation of the graph. (b) The line y = 2 meets the graph at points P & Q. Find the coordinates of these two points. Reveal answer only Go to full solution Go to Marker’s Comments Go to Trig Graphs & Equations Menu EXIT

98 TRIG GRAPHS & EQUATIONS : Question 3
The diagram shows a the graph of a sine function from 0 to 2/3. y = 2 P Q 2/3 (a) State the equation of the graph. (b) The line y = 2 meets the graph at points P & Q. Find the coordinates of these two points. Graph is y = 4sin3x Reveal answer only P is (/18, 2) and Q is (5/18, 2). Go to full solution Go to Marker’s Comments Go to Trig Graphs & Equations Menu EXIT

99 Question 3 One complete wave from 0 to 2/3 so 3 waves from 0 to 2.
The diagram shows a the graph of a sine function from 0 to 2/3. (a) State the equation of the graph. Max/min = ±4 4sin(…) 2/3 P Q y = 2 Graph is y = 4sin3x Begin Solution Continue Solution Continue Solution Markers Comments Trig Graphs etc. Menu Back to Home

100 Question 3 Graph is y = 4sin3x (b) The line y = 2 meets the graph
at points P & Q. Find the coordinates of these two points. (b) At P & Q y = 4sin3x and y = 2 so 4sin3x = 2 or sin3x = 1/2 Q1 or Q2 2/3 P Q y = 2  -    +  2 -  sin all tan cos sin-1(1/2) = /6 Q1: angle = /6 so 3x = /6 1 2 3 /6 ie x = /18 Q2: angle =  - /6 Begin Solution so 3x = 5/6 Continue Solution ie x = 5/18 Markers Comments P is (/18, 2) and Q is (5/18, 2). Trig Graphs etc. Menu Back to Home

101 Identify graph is of the form y = asinbx.
Markers Comments Identify graph is of the form y = asinbx. Must justify choice of a and b. Possible justification of a One complete wave from 0 to 2/3 so 3 waves from 0 to 2. Max/min = ±4 4sin(…) Max = 4, Min = sin(…) y = sinx stretched by a factor of 4 Graph is y = 4sin3x Possible justification for b Period = 3 waves from 0 to Next Comment Trig Graphs Menu Back to Home

102 Use the quadrant rule to find the solutions. or sin3x = 1/2 Q1 or Q2
Markers Comments At intersection y1 = y2 4sin3x = 2 Solve for sin3x Graph is y = 4sin3x (b) At P & Q y = 4sin3x and y = 2 so 4sin3x = 2 Use the quadrant rule to find the solutions. or sin3x = 1/2 Q1 or Q2  -    +  2 -  sin all tan cos sin-1(1/2) = /6 Must learn special angles or be able to calculate from triangles. Q1: angle = /6 1 45° 2 60° 30° so 3x = /6 1 2 3 /6 ie x = /18 Q2: angle =  - /6 Take care to state coordinates. so 3x = 5/6 Next Comment ie x = 5/18 Trig Graphs Menu P is (/18, 2) and Q is (5/18, 2). Back to Home

103 HIGHER – ADDITIONAL QUESTION BANK
You have chosen to study: UNIT 1 : Functions & Graphs Please choose a question to attempt from the following: 1 2 3 4 Back to Unit 1 Menu EXIT

104 FUNCTIONS & GRAPHS : Question 1
This graph shows the the function y = g(x). Make a sketch of the graph of the function y = 4 – g(-x). y = g(x) -8 12 (-p,q) (u,-v) Reveal answer only Go to full solution Go to Marker’s Comments Go to Functions & Graphs Menu Go to Main Menu EXIT

105 FUNCTIONS & GRAPHS : Question 1
This graph shows the the function y = g(x). Make a sketch of the graph of the function y = 4 – g(-x). y = g(x) -8 12 (-p,q) (u,-v) Reveal answer only (p,-q+4) (8,4) (-12,4) (0,4) (-u,v+4) y = 4 – g(-x) Go to full solution Go to Marker’s Comments Go to Functions & Graphs Menu Go to Main Menu EXIT

106 Question 1 y = 4 – g(-x) = -g(-x) + 4
This graph shows the the function y = g(x). Make a sketch of the graph of the function y = 4 – g(-x). A B Reflect in X-axis C Slide 4 up Reflect in Y-axis y = g(x) -8 12 (-p,q) (u,-v) Known Points (-8,0), (-p,q), (0,0), (u,-v), (12,0) A (-8,0), (-p,-q), (0,0), (u,v), (12,0) B Begin Solution (8,0), (p,-q), (0,0), (-u,v), (-12,0) Continue Solution C Markers Comments (8,4), (p,-q+4), (0,4), (-u,v+4), (-12,4) Functions & Graphs Menu Back to Home

107 Question 1 (8,4), (p,-q+4), (0,4), (-u,v+4), (-12,4)
This graph shows the the function y = g(x). Make a sketch of the graph of the function y = 4 – g(-x). Now plot points and draw curve through them. (p,-q+4) (8,4) (-12,4) (0,4) (-u,v+4) y = 4 – g(-x) y = g(x) -8 12 (-p,q) (u,-v) Begin Solution Continue Solution Markers Comments Functions & Graphs Menu Back to Home

108 Change order to give form: y = k.g(x) + c y = 4 – g(-x) = -g(-x) + 4
Markers Comments Change order to give form: y = k.g(x) + c y = 4 – g(-x) = -g(-x) + 4 When the function is being changed by more than one related function take each change one at a time either listing the coordinates or sketching the steps to final solution. A B Reflect in X-axis C Slide 4 up Reflect in Y-axis Known Points (-8,0), (-p,q), (0,0), (u,-v), (12,0) A (-8,0), (-p,-q), (0,0), (u,v), (12,0) B (8,0), (p,-q), (0,0), (-u,v), (-12,0) Next Comment C Functions Menu (8,4), (p,-q+4), (0,4), (-u,v+4), (-12,4) Back to Home

109 Slide k units parallel to y-axis kf(x) Stretch by a factor = k -f(x)
Markers Comments y = 4 – g(-x) = -g(-x) + 4 A Learn Rules: Not given on formula sheet B Reflect in X-axis C f(x) + k Slide k units parallel to y-axis kf(x) Stretch by a factor = k -f(x) Reflect in the x-axis f(x-k) Slide k units parallel to the x-axis f(-x) Reflect in y-axis Slide 4 up Reflect in Y-axis Known Points (-8,0), (-p,q), (0,0), (u,-v), (12,0) A (-8,0), (-p,-q), (0,0), (u,v), (12,0) B (8,0), (p,-q), (0,0), (-u,v), (-12,0) Next Comment C Functions Menu (8,4), (p,-q+4), (0,4), (-u,v+4), (-12,4) Back to Home

110 Now plot points and draw curve through them.
Markers Comments In any curve sketching question use a ruler and annotate the sketch i.e. label all known coordinates. (8,4), (p,-q+4), (0,4), (-u,v+4), (-12,4) Now plot points and draw curve through them. (p,-q+4) (8,4) (-12,4) (0,4) (-u,v+4) y = 4 – g(-x) Next Comment Functions Menu Back to Home

111 FUNCTIONS & GRAPHS : Question 2
y = ax (1,a) This graph shows the the function y = ax. Make sketches of the graphs of the functions (I) y = a(x+2) (II) y = 2ax - 3 Reveal answer only Go to full solution Go to Marker’s Comments Go to Functions & Graphs Menu Go to Main Menu EXIT

112 FUNCTIONS & GRAPHS : Question 2
ANSWER TO PART (I) This graph shows the the function y = ax. Make sketches of the graphs of the functions (I) y = a(x+2) (II) y = 2ax - 3 (-1,a) (-2,1) y = a(x+2) y = ax Reveal answer only ANSWER to PART (II) Go to full solution y = 2ax - 3 y = ax (0,-1) (1,2a-3) Go to Marker’s Comments Go to Functions & Graphs Menu EXIT

113 Question 2 (I) y = a(x+2) Make sketches of the graphs of the functions
f(x) = ax so a(x+2) = f(x+2) y = ax (1,a) move f(x) 2 to left (0,1)(-2,1) & (1,a) (-1,a) (-1,a) (-2,1) y = a(x+2) y = ax Begin Solution Continue Solution Markers Comments Functions & Graphs Menu Back to Home

114 Question 2 (II) y = 2ax - 3 Make sketches of the graphs of the functions (II) y = 2ax - 3 f(x) = ax so 2ax - 3 = 2f(x) - 3 y = ax (1,a) double y-coords slide 3 down (0,1)(0,2)(0,-1) (1,a) (1,2a) (1,2a-3) y = 2ax - 3 y = ax (0,-1) (1,2a-3) Begin Solution Continue Solution Markers Comments Functions & Graphs Menu Back to Home

115 When the problem is given in terms of a specific function
Markers Comments When the problem is given in terms of a specific function rather in terms of the general f(x), change back to f(x) eg. y = 2x, y = log3x, y = x2 + 3x, each becomes y = f(x) (I) y = a(x+2) f(x) = ax so a(x+2) = f(x+2) move f(x) 2 to left In any curve sketching question use a ruler and annotate the sketch i.e. label all known coordinates. (0,1)(-2,1) & (1,a) (-1,a) (-1,a) (-2,1) y = a(x+2) y = ax Next Comment Functions Menu Back to Home

116 Slide k units parallel to y-axis kf(x) Stretch by a factor = k -f(x)
Markers Comments (I) y = a(x+2) Learn Rules: Not given on formula sheet f(x) = ax so a(x+2) = f(x+2) f(x) + k Slide k units parallel to y-axis kf(x) Stretch by a factor = k -f(x) Reflect in the x-axis f(x-k) Slide k units parallel to the x-axis f(-x) Reflect in y-axis move f(x) 2 to left (0,1)(-2,1) & (1,a) (-1,a) (-1,a) (-2,1) y = a(x+2) y = ax Next Comment Functions Menu Back to Home

117 When the problem is given in terms of a specific function
Markers Comments When the problem is given in terms of a specific function rather in terms of the general f(x), change back to f(x) eg. y = 2x, y = log3x, y = x2 + 3x, each becomes y = f(x) (II) y = 2ax - 3 f(x) = ax so 2ax - 3 = 2f(x) - 3 double y-coords slide 3 down In any curve sketching question use a ruler and annotate the sketch i.e. label all known coordinates. (0,1)(0,2)(0,-1) (1,a) (1,2a) (1,2a-3) y = 2ax - 3 y = ax (0,-1) (1,2a-3) Next Comment Functions Menu Back to Home

118 Slide k units parallel to y-axis kf(x) Stretch by a factor = k -f(x)
Markers Comments (II) y = 2ax - 3 Learn Rules: Not given on formula sheet f(x) = ax so 2ax - 3 = 2f(x) - 3 f(x) + k Slide k units parallel to y-axis kf(x) Stretch by a factor = k -f(x) Reflect in the x-axis f(x-k) Slide k units parallel to the x-axis f(-x) Reflect in y-axis double y-coords slide 3 down (0,1)(0,2)(0,-1) (1,a) (1,2a) (1,2a-3) y = 2ax - 3 y = ax (0,-1) (1,2a-3) Next Comment Functions Menu Back to Home

119 FUNCTIONS & GRAPHS : Question 3
Two functions f and g are defined on the set of real numbers by the formulae f(x) = 2x and g(x) = x2 . (a) Find formulae for (i) f(g(x)) (ii) g(f(x)) . (b) Hence show that the equation g(f(x)) = f(g(x)) has only one real solution. Reveal answer only Go to full solution Go to Marker’s Comments Go to Functions & Graphs Go to Main Menu EXIT

120 FUNCTIONS & GRAPHS : Question 3
Two functions f and g are defined on the set of real numbers by the formulae f(x) = 2x and g(x) = x2 . (a) Find formulae for (i) f(g(x)) (ii) g(f(x)) . (b) Hence show that the equation g(f(x)) = f(g(x)) has only one real solution. (a) (i) = 2x2 - 1 Reveal answer only Go to full solution = 4x2 – 4x + 1 (ii) Go to Marker’s Comments Go to Functions & Graphs Go to Main Menu EXIT

121 Question 3 (a)(i) f(g(x)) Two functions f and g are defined
on the set of real numbers by the formulae f(x) = 2x and g(x) = x2 . = f(x2) = 2x2 - 1 (ii) g(f(x)) Find formulae for (i) f(g(x)) (ii) g(f(x)) . = g(2x-1) = (2x – 1)2 = 4x2 – 4x + 1 Begin Solution Continue Solution Continue Solution Markers Comments Functions & Graphs Menu Back to Home

122 Question 3 f(g(x)) = 2x2 - 1 Two functions f and g are defined
on the set of real numbers by the formulae f(x) = 2x and g(x) = x2 . g(f(x)) = 4x2 – 4x + 1 (b) g(f(x)) = f(g(x)) 4x2 – 4x + 1 = 2x2 - 1 2x2 – 4x + 2 = 0 Hence show that the equation g(f(x)) = f(g(x)) has only one real solution. x2 – 2x + 1 = 0 (x – 1)(x – 1) = 0 x = 1 Hence only one real solution! Begin Solution Continue Solution Markers Comments Functions & Graphs Menu Back to Home

123 In composite function problems take at least 3 lines to
Markers Comments (a) In composite function problems take at least 3 lines to answer the problem: State required composite function: f(g(x)) Replace g(x) without simplifying: f(x2) In f(x) replace each x by g(x): x2 - 1 (a)(i) f(g(x)) = f(x2) = 2x2 - 1 (ii) g(f(x)) (II) State required composite function: g(f(x)) Replace f(x) without simplifying: g(2x-1) In g(x) replace each x by f(x): = g(2x-1) = (2x – 1)2 = 4x2 – 4x + 1 (2x – 1)2 Next Comment Functions Menu Back to Home

124 simplify and factorise. f(g(x)) = 2x2 - 1
Markers Comments (b) Only one way to solve resulting equation: Terms to the left, simplify and factorise. f(g(x)) = 2x2 - 1 g(f(x)) = 4x2 – 4x + 1 (b) g(f(x)) = f(g(x)) 4x2 – 4x + 1 = 2x2 - 1 2x2 – 4x + 2 = 0 x2 – 2x + 1 = 0 (x – 1)(x – 1) = 0 x = 1 Hence only one real solution! Next Comment Functions Menu Back to Home

125 FUNCTIONS & GRAPHS : Question 4
A function g is defined by the formula g(x) = (x1) (x – 1) (a) Find a formula for h(x) = g(g(x)) in its simplest form. (b) State a suitable domain for h. Reveal answer only Go to full solution Go to Marker’s Comments Go to Functions & Graphs Menu Go to Main Menu EXIT

126 FUNCTIONS & GRAPHS : Question 4
A function g is defined by the formula g(x) = (x1) (x – 1) (a) Find a formula for h(x) = g(g(x)) in its simplest form. (b) State a suitable domain for h. = (2x - 2) (3 – x) h(x) Reveal answer only Go to full solution Domain = {x  R: x  3} Go to Marker’s Comments Go to Functions & Graphs Menu Go to Main Menu EXIT

127 Question 4 = 2 - 1 = 2 = 2 (a) g(g(x)) = g( )
A function g is defined by the formula g(x) = (x1) (x – 1) = (x – 1) - 1 = Find a formula for h(x) = g(g(x)) in its simplest form. 2 - (x – 1) (x – 1) = (3 - x) .(x – 1) (x - 1) .(3 – x) = 2 Begin Solution = (2x - 2) (3 – x) Continue Solution Markers Comments Continue Solution Functions & Graphs Menu Back to Home

128 Question 4 = (2x - 2) . . (3 – x) h(x) A function g is defined by the
formula g(x) = (x1) (x – 1) (b) For domain x  0 Find a formula for h(x) = g(g(x)) in its simplest form. Domain = {x  R: x  3} (b) State a suitable domain for h. Begin Solution Continue Solution Markers Comments Functions & Graphs Menu Back to Home

129 = 2 - 1 = 2 - 1 = 2 Markers Comments (a)
In composite function problems take at least 3 lines to answer the problem: State required composite function: g(g(x)) Replace g(x) without simplifying: g(2/(x-1)) In g(x) replace each x by g(x): (x – 1) (a) g(g(x)) = g( ) = (x – 1) - 1 = 2 (x-1) - 1 2 - (x – 1) (x – 1) = (3 - x) .(x – 1) (x - 1) .(3 – x) = 2 = (2x - 2) (3 – x) Next Comment Functions Menu Back to Home

130 In finding a suitable domain it is often necessary to restrict R
Markers Comments (b) In finding a suitable domain it is often necessary to restrict R to prevent either division by zero or the root of a negative number: In this case: x = 0 i.e. preventing division by zero. = (2x - 2) (3 – x) h(x) (b) For domain x  0 Domain = {x  R: x  3} Next Comment Functions Menu Back to Home

131 Polynomials Quadratics Integration Addition Formulae The Circle
HIGHER – ADDITIONAL QUESTION BANK UNIT 2 : Polynomials Quadratics Integration Addition Formulae The Circle EXIT

132 HIGHER – ADDITIONAL QUESTION BANK
You have chosen to study: UNIT 2 : Polynomials Please choose a question to attempt from the following: 1 2 3 4 Back to Unit 2 Menu EXIT

133 POLYNOMIALS : Question 1
Show that x = 3 is a root of the equation x3 + 3x2 – 10x – 24 = 0. Hence find the other roots. Reveal answer only Go to full solution Go to Marker’s Comments Go to Polynomial Menu Go to Main Menu EXIT

134 POLYNOMIALS : Question 1
Show that x = 3 is a root of the equation x3 + 3x2 – 10x – 24 = 0. Hence find the other roots. other roots are x = -4 & x = -2 Reveal answer only Go to full solution Go to Marker’s Comments Go to Polynomial Menu Go to Main Menu EXIT

135 Question 1 18 Using the nested method -
coefficients are 1, 3, -10, -24 Show that x = 3 is a root of the equation x3 + 3x2 – 10x – 24 = 0. f(3) = 3 Hence find the other roots. 18 3 24 1 6 8 f(3) = 0 so x = 3 is a root. Also (x – 3) is a factor. Other factor: x2 + 6x + 8 or (x + 4)(x + 2) Begin Solution If (x + 4)(x + 2) = 0 then x = -4 or x = -2 Continue Solution Markers Comments Hence other roots are x = -4 & x = -2 Polynomial Menu Back to Home

136 State clearly in solution that f(3) = 0 x = 3 is a root
Markers Comments State clearly in solution that f(3) = x = 3 is a root Using the nested method - coefficients are 1, 3, -10, -24 Show completed factorisation of cubic i.e. (x - 3)(x + 4)(x + 2) = 0 f(3) = 3 18 3 24 1 6 8 Take care to set factorised expression = 0 f(3) = 0 so x = 3 is a root. Also (x – 3) is a factor. List all the roots of the polynomial x = 3, x = -4, x = -2 Other factor: x2 + 6x + 8 or (x + 4)(x + 2) If (x + 4)(x + 2) = 0 then x = -4 or x = -2 Next Comment Polynomial Menu Hence other roots are x = -4 & x = -2 Back to Home

137 POLYNOMIALS : Question 2
Given that (x + 4) is a factor of the polynomial f(x) = 3x3 + 8x2 + kx find the value of k. Hence solve the equation 3x3 + 8x2 + kx + 4 = 0 for this value of k. Reveal answer only Go to full solution Go to Marker’s Comments Go to Polynomial Menu Go to Main Menu EXIT

138 POLYNOMIALS : Question 2
Given that (x + 4) is a factor of the polynomial f(x) = 3x3 + 8x2 + kx find the value of k. Hence solve the equation 3x3 + 8x2 + kx + 4 = 0 for this value of k. k = -15 Reveal answer only So full solution of equation is x = -4 or x = 1/3 or x = 1 Go to full solution Go to Marker’s Comments Go to Polynomial Menu Go to Main Menu EXIT

139 Question 2 Since (x + 4) a factor then f(-4) = 0 .
Given that (x + 4) is a factor of the polynomial f(x) = 3x3 + 8x2 + kx + 4 find the value of k. Hence solve the equation 3x3 + 8x2 + kx + 4 = 0 for this value of k. Now using the nested method - coefficients are 3, 8, k, 4 f(-4) = -4 3 8 k -12 16 (-4k – 64) (-4k – 60) 3 -4 (k + 16) Since k – 60 = 0 then k = 60 Begin Solution Continue Solution so k = -15 Markers Comments Polynomial Menu Back to Home

140 Question 2 If k = -15 then we now have
Given that (x + 4) is a factor of the polynomial f(x) = 3x3 + 8x2 + kx + 4 find the value of k. Hence solve the equation 3x3 + 8x2 + kx + 4 = 0 for this value of k. f(-4) = -4 -12 16 -4 Other factor is x2 – 4x + 1 or (3x - 1)(x – 1) If (3x - 1)(x – 1) = 0 then x = 1/3 or x = 1 Begin Solution So full solution of equation is: x = -4 or x = 1/3 or x = 1 Continue Solution Markers Comments Polynomial Menu Back to Home

141 The working in the nested solution can sometimes be
Markers Comments The working in the nested solution can sometimes be eased by working in both directions toward the variable: Since (x + 4) a factor then f(-4) = 0 . Now using the nested method - coefficients are 3, 8, k, 4 k 4 f(-4) = -4 3 8 k -12 16 (-4k – 64) (-4k – 60) 3 -4 (k + 16) k + 16 = 1 k = -15 Since k – 60 = 0 then k = 60 Next Comment so k = -15 Polynomial Menu Back to Home

142 Simply making f(-4) = 0 will also yield k i.e.
Markers Comments Simply making f(-4) = 0 will also yield k i.e. 3(-4)3 + 8(-4)2 + k(-4) + 4 = 0 k = -15 Since (x + 4) a factor then f(-4) = 0 . Now using the nested method - coefficients are 3, 8, k, 4 f(-4) = -4 3 8 k -12 16 (-4k – 64) (-4k – 60) 3 -4 (k + 16) Since k – 60 = 0 then k = 60 Next Comment so k = -15 Polynomial Menu Back to Home

143 Show completed factorisation of the cubic: (x + 4)(3x - 1)(x - 1) = 0
Markers Comments Show completed factorisation of the cubic: (x + 4)(3x - 1)(x - 1) = 0 If k = then we now have f(-4) = -4 -12 16 -4 Other factor is x2 – 4x + 1 or (3x - 1)(x – 1) If (3x - 1)(x – 1) = 0 So full solution of equation is: x = -4 or x = 1/3 or x = 1 Next Comment Polynomial Menu Back to Home

144 POLYNOMIALS : Question 3
Given that f(x) = 6x3 + 13x show that (x + 2) is a factor of f(x). Hence express f(x) in its fully factorised form. Reveal answer only Go to full solution Go to Marker’s Comments Go to Polynomial Menu Go to Main Menu EXIT

145 POLYNOMIALS : Question 3
Given that f(x) = 6x3 + 13x show that (x + 2) is a factor of f(x). Hence express f(x) in its fully factorised form. 6x3 + 13x = (3x + 2)(2x - 1)(x + 2) Reveal answer only Go to full solution Go to Marker’s Comments Go to Polynomial Menu Go to Main Menu EXIT

146 Question 3 Using the nested method - coefficients are 6, 13, 0, -4
Given that f(x) = 6x3 + 13x2 - 4 show that (x + 2) is a factor of f(x). Hence express f(x) in its fully factorised form. f(-2) = -2 -12 -2 4 6 1 -2 f(-2) = 0 so (x + 2) is a factor Begin Solution Continue Solution Markers Comments Polynomial Menu Back to Home

147 Question 3 Using the nested method - coefficients are 6, 13, 0, -4
Given that f(x) = 6x3 + 13x2 - 4 show that (x + 2) is a factor of f(x). Hence express f(x) in its fully factorised form. f(-2) = -2 -12 -2 4 6 1 -2 Other factor is x2 + x – 2 or (3x + 2)(2x - 1) Hence 6x3 + 13x2 - 4 = (3x + 2)(2x - 1)(x + 2) Begin Solution Continue Solution Markers Comments Polynomial Menu Back to Home

148 State clearly in solution that f(-2) = 0 x = -2 is a root
Markers Comments State clearly in solution that f(-2) = x = -2 is a root Using the nested method - coefficients are 6, 13, 0, -4 f(-2) = -2 -12 -2 4 6 1 -2 f(-2) = 0 so (x + 2) is a factor Next Comment Polynomial Menu Back to Home

149 Show completed factorisation of cubic i.e. (3x + 2)(2x - 1)(x +2).
Markers Comments Show completed factorisation of cubic i.e. (3x + 2)(2x - 1)(x +2). Using the nested method - coefficients are 6, 13, 0, -4 f(-2) = -2 -12 -2 4 Can show (x + 2) is a factor by showing f(-2) = 0 but still need nested method for quadratic factor. 6 1 -2 Other factor is x2 + x – 2 or (3x + 2)(2x - 1) Hence 6x3 + 13x2 - 4 = (3x + 2)(2x - 1)(x + 2) Next Comment Polynomial Menu Back to Home

150 POLYNOMIALS : Question 4
A busy road passes through several small villages so it is decided to build a by-pass to reduce the volume of traffic. Relative to a set of coordinate axes the road can be modelled by the curve y = -x3 + 6x2 – 3x – 10. The by-pass is a tangent to this curve at point P and rejoins the original road at Q as shown below. P Q y = -x3 + 6x2 – 3x – 10 4 bypass Find the coordinates of P and the equation of the bypass PQ. Hence find the coordinates of Q – the point where the bypass rejoins the original road. EXIT Reveal answer only Go to full solution

151 POLYNOMIALS : Question 4
A busy road passes through several small villages so it is decided to build a by-pass to reduce the volume of traffic. Relative to a set of coordinate axes the road can be modelled by the curve y = -x3 + 6x2 – 3x – 10. The by-pass is a tangent to this curve at point P and rejoins the original road at Q as shown below. Find the coordinates of P and the equation of the bypass PQ. Hence find the coordinates of Q – the point where the bypass rejoins the original road. P is (4,10) PQ is y = -3x + 22 Q is (-2,28) Go to full solution Go to Marker’s Comments Go to Polynomial Menu Go to Main Menu EXIT

152 Question 4 At point P, x = 4 so using the
equation of the curve we get ….. y = -x3 + 6x2 – 3x – 10 Find the coordinates of P and the equation of the bypass PQ. y = (6 X 42) – (3 X 4) - 10 = – P Q y = -x3 + 6x2 – 3x – 10 4 = 10 ie P is (4,10) Gradient of tangent = gradient of curve = dy/dx = -3x2 + 12x - 3 When x = 4 then dy/dx = (-3 X 16) + (12 X 4) – 3 Begin Solution = – 3 = -3 Continue Solution Markers Comments Polynomial Menu Back to Home

153 Question 4 P is (4,10) Find the coordinates of P
dy/dx = -3 y = -x3 + 6x2 – 3x – 10 Find the coordinates of P and the equation of the bypass PQ. Now using : y – b = m(x – a) where (a,b) = (4,10) & m = -3 P Q y = -x3 + 6x2 – 3x – 10 4 We get y – 10 = -3(x – 4) or y – 10 = -3x + 12 So PQ is y = -3x + 22 Begin Solution Continue Solution Markers Comments Polynomial Menu Back to Home

154 Question 4 (b) The tangent & curve meet whenever
y = -x3 + 6x2 – 3x – 10 y = -3x and y = -x3 + 6x2 – 3x – 10 (b)Hence find the coordinates of Q – the point where the bypass rejoins the original road. ie x + 22 = -x3 + 6x2 – 3x – 10 or x3 - 6x = 0 P Q y = -x3 + 6x2 – 3x – 10 4 PQ is y = -3x + 22 We already know that x = 4 is one solution to this so using the nested method we get ….. f(4) = 4 4 -8 -32 Begin Solution 1 -2 -8 Continue Solution Markers Comments Other factor is x2 – 2x - 8 Polynomial Menu Back to Home

155 Question 4 (b) The other factor is x2 – 2x - 8 = (x – 4)(x + 2)
y = -x3 + 6x2 – 3x – 10 = (x – 4)(x + 2) (b)Hence find the coordinates of Q – the point where the bypass rejoins the original road. Solving (x – 4)(x + 2) = 0 we get x = 4 or x = -2 P Q y = -x3 + 6x2 – 3x – 10 4 PQ is y = -3x + 22 It now follows that Q has an x-coordinate of -2 Using y = -3x if x = -2 then y = = 28 Begin Solution Hence Q is (-2,28) Continue Solution Markers Comments Polynomial Menu Back to Home

156 Must use differentiation to find gradient. Learn rule:
Markers Comments (a) Must use differentiation to find gradient. Learn rule: “Multiply by the power then reduce the power by 1” At point P, x = 4 so using the equation of the curve we get ….. y = (6 X 42) – (3 X 4) - 10 = – = 10 ie P is (4,10) Gradient of tangent = gradient of curve = dy/dx = -3x2 + 12x - 3 When x = 4 then dy/dx = (-3 X 16) + (12 X 4) – 3 Next Comment = – 3 = -3 Polynomial Menu Back to Home

157 the point of contact (4,10)& Gradient of curve at this
Markers Comments (a) P is (4,10) Use : the point of contact (4,10)& Gradient of curve at this point (m = -3) in equation y - b = m(x - a) dy/dx = -3 Now using : y – b = m(x – a) where (a,b) = (4,10) & m = -3 We get y – 10 = -3(x – 4) or y – 10 = -3x + 12 So PQ is y = -3x + 22 Next Comment Polynomial Menu Back to Home

158 simplify and factorise
Markers Comments (b) (b) The tangent & curve meet whenever At intersection y1 = y2 y = -3x and y = -x3 + 6x2 – 3x – 10 Terms to the left, simplify and factorise ie x + 22 = -x3 + 6x2 – 3x – 10 or x3 - 6x = 0 We already know that x = 4 is one solution to this so using the nested method we get ….. f(4) = 4 4 -8 -32 1 -2 -8 Next Comment Polynomial Menu Other factor is x2 – 2x - 8 Back to Home

159 Note solution x = 4 appears twice: Repeated root tangency
Markers Comments (b) other factor is x2 – 2x - 8 Note solution x = 4 appears twice: Repeated root tangency = (x – 4)(x + 2) Solving (x – 4)(x + 2) = 0 we get x = 4 or x = -2 It now follows that Q has an x-coordinate of -2 Using y = -3x if x = -2 then y = = 28 Hence Q is (-2,28) Next Comment Polynomial Menu Back to Home

160 HIGHER – ADDITIONAL QUESTION BANK
You have chosen to study: UNIT 2 : Quadratics Please choose a question to attempt from the following: 1 2 3 4 5 6 Back to Unit 2 Menu EXIT

161 QUADRATICS : Question 1 (a) Express f(x) = x2 – 8x in the form (x – a)2 + b. (b) Hence, or otherwise, sketch the graph of y = f(x). Reveal answer only Go to full solution Go to Marker’s Comments Go to Quadratics Menu Go to Main Menu EXIT

162 QUADRATICS : Question 1 (a) Express f(x) = x2 – 8x in the form (x – a)2 + b. (b) Hence, or otherwise, sketch the graph of y = f(x). = (x – 4)2 + 5 (a) Reveal answer only y = x2 – 8x + 21 (b) Go to full solution Go to Marker’s Comments Go to Quadratics Menu Go to Main Menu (4,5) EXIT

163 Question 1 (a) f(x) = x2 – 8x + 21 Express f(x) = x2 – 8x + 21
in the form (x – a)2 + b. Hence, or otherwise, sketch the graph of y = f(x). = (x2 – 8x + ) + 21 (-82)2 = (x – 4)2 + 5 Begin Solution Continue Solution Markers Comments Quadratics Menu Back to Home

164 Question 1 f(x) = (x – 4)2 + 5 has a minimum
value of 5 when (x – 4)2 = 0 ie x = 4 Express f(x) = x2 – 8x + 21 in the form (x – a)2 + b. Hence, or otherwise, sketch the graph of y = f(x). So the graph has a minimum turning point at (4,5). When x = 0 , y = 21 (from original formula!) so Y-intercept is (0,21). Begin Solution Continue Solution Markers Comments Quadratics Menu Back to Home

165 Question 1 Graph looks like…. Express f(x) = x2 – 8x + 21
in the form (x – a)2 + b. Hence, or otherwise, sketch the graph of y = f(x). y = x2 – 8x + 21 (0,21) (4,5) Begin Solution Continue Solution Markers Comments Quadratics Menu Back to Home

166 Move towards desired form in stages: f(x) = x2 - 8x + 21
Markers Comments Move towards desired form in stages: f(x) = x2 - 8x + 21 = (x2 - 8x) + 21 = (x2 - 8x +16) (a) f(x) = x2 – 8x + 21 = (x2 – 8x + ) + 21 (-82)2 = (x – 4)2 + 5 Find the number to complete the perfect square and balance the expression. (a + b) 2 = a2 + 2ab + b2 = (x - 4)2 + 5 Next Comment Quadratics Menu Back to Home

167 The sketch can also be obtained by calculus:
Markers Comments The sketch can also be obtained by calculus: f(x) = (x – 4) has a minimum value of 5 when (x – 4)2 = 0 ie x = 4 So the graph has a minimum turning point at (4,5). When x = 0 , y = 21 (from original formula!) so Y-intercept is (0,21). (0,21) Next Comment Quadratics Menu (4,5) Back to Home

168 coefficient of x2 is positive.
Markers Comments Min. Turning Point (4,5), coefficient of x2 is positive. f(x) = (x – 4) has a minimum value of 5 when (x – 4)2 = 0 ie x = 4 So the graph has a minimum turning point at (4,5). Hence sketch. 21 (4,5) When x = 0 , y = 21 (from original formula!) so Y-intercept is (0,21). (0,21) Next Comment Quadratics Menu (4,5) Back to Home

169 QUADRATICS : Question 2 Express f(x) = 7 + 8x - 4x2 in the form a - b(x - c)2. (b) Hence find the maximum turning point on the graph of y = f(x). Reveal answer only Go to full solution Go to Marker’s Comments Go to Quadratics Menu Go to Main Menu EXIT

170 QUADRATICS : Question 2 Express f(x) = 7 + 8x - 4x2 in the form a - b(x - c)2. (b) Hence find the maximum turning point on the graph of y = f(x). (a) = (x – 1)2 Reveal answer only Go to full solution Go to Marker’s Comments maximum t p is at (1,11) . (b) Go to Quadratics Menu Go to Main Menu EXIT

171 Question 2 (a) f(x) = 7 + 8x - 4x2 Express f(x) = 7 + 8x - 4x2 in
the form a - b(x - c)2. Hence find the maximum turning point on the graph of y = f(x). = - 4x2 + 8x + 7 = -4[x2 – 2x] + 7 = -4[(x2 – 2x + ) ] + 7 (-22)2 = -4[(x – 1) ] + 7 = -4(x – 1) = (x – 1)2 Begin Solution Continue Solution Markers Comments Quadratics Menu Back to Home

172 Question 2 = 11 - 4(x – 1)2 Express f(x) = 7 + 8x - 4x2 in
the form a - b(x - c)2. Hence find the maximum turning point on the graph of y = f(x). Maximum value is 11 when (x – 1)2 = 0 ie x = 1. so maximum turning point is at (1,11) . Begin Solution Continue Solution Markers Comments Quadratics Menu Back to Home

173 Move towards desired form in stages: f(x) = 7 + 8x - 4x2
Markers Comments Move towards desired form in stages: f(x) = 7 + 8x - 4x2 = - 4x2 + 8x + 7 = (- 4x2 + 8x) + 7 (a) f(x) = x - 4x2 = - 4x2 + 8x + 7 = -4[x2 – 2x] + 7 = -4[(x2 – 2x + ) ] + 7 (-22)2 Must reduce coefficient of x2 to 1 = -4(x2 - 2x) + 7 = -4[(x – 1) ] + 7 = -4(x – 1) = (x – 1)2 Next Comment Quadratics Menu Back to Home

174 Must reduce coefficient of x2 to 1 = -4(x2 - 2x) + 7
Markers Comments Must reduce coefficient of x2 to 1 = -4(x2 - 2x) + 7 (a) f(x) = x - 4x2 = - 4x2 + 8x + 7 = -4[x2 – 2x] + 7 Find the number to complete the perfect square and balance the expression (a + b) 2 = a2 + 2ab + b2 = -4[(x2 – 2x + ) ] + 7 (-22)2 = -4[(x – 1) ] + 7 = -4[(x2 - 2x +1) -1] +7 ) = -4(x-1) = (x-1)2 Max. TP at (1,11) = -4(x – 1) = (x – 1)2 Maximum value is 11 when (x – 1)2 = 0 ie x = 1. Next Comment Quadratics Menu so maximum turning point is at (1,11) . Back to Home

175 QUADRATICS : Question 3 For what value(s) of k does the equation 4x2 – kx + (k + 5) = 0 have equal roots. Reveal answer only Go to full solution Go to Marker’s Comments Go to Quadratics Menu Go to Main Menu EXIT

176 QUADRATICS : Question 3 For what value(s) of k does the equation 4x2 – kx + (k + 5) = 0 have equal roots. Reveal answer only k = -4 or k = 20 Go to full solution Go to Marker’s Comments Go to Quadratics Menu Go to Main Menu EXIT

177 Question 3 Let 4x2 – kx + (k + 5) = ax2 + bx + c
For what value(s) of k does the equation 4x2 – kx + (k + 5) = 0 have equal roots. then a = 4, b = -k & c = (k + 5) For equal roots we need discriminant = 0 ie b2 – 4ac = 0 (-k)2 – (4 X 4 X (k + 5)) = 0 k2 – 16(k + 5) = 0 k2 – 16k = 0 (k + 4)(k – 20) = 0 k = -4 or k = 20 Begin Solution Continue Solution Markers Comments Quadratics Menu Back to Home

178 Learn Rules relating to discriminant b2- 4ac b2- 4ac = 0 Equal roots
Markers Comments Learn Rules relating to discriminant b2- 4ac b2- 4ac = Equal roots b2- 4ac > Real, distinct roots b2- 4ac < No real roots b2- 4ac Real roots, equal or distinct Let 4x2 – kx + (k + 5) = ax2 + bx + c then a = 4, b = -k & c = (k + 5) For equal roots we need discriminant = 0 ie b2 – 4ac = 0 (-k)2 – (4 X 4 X (k + 5)) = 0 k2 – 16(k + 5) = 0 k2 – 16k = 0 (k + 4)(k – 20) = 0 k = -4 or k = 20 Next Comment Quadratics Menu Back to Home

179 Must use factorisation to solve resulting quadratic.
Markers Comments Must use factorisation to solve resulting quadratic. Trial and error receives no credit. Let 4x2 – kx + (k + 5) = ax2 + bx + c then a = 4, b = -k & c = (k + 5) For equal roots we need discriminant = 0 ie b2 – 4ac = 0 (-k)2 – (4 X 4 X (k + 5)) = 0 k2 – 16(k + 5) = 0 k2 – 16k = 0 (k + 4)(k – 20) = 0 k = -4 or k = 20 Next Comment Quadratics Menu Back to Home

180 QUADRATICS : Question 4 The equation of a parabola is f(x) = px2 + 5x – 2p . Prove that the equation f(x) = 0 always has two distinct roots. Reveal answer only Go to full solution Go to Marker’s Comments Go to Quadratics Menu Go to Main Menu EXIT

181 QUADRATICS : Question 4 The equation of a parabola is f(x) = px2 + 5x – 2p . Prove that the equation f(x) = 0 always has two distinct roots. discriminant = b2 – 4ac Reveal answer only = 8p Go to full solution Since p2  0 for all values of p then 8p > 0. Go to Marker’s Comments The discriminant is always positive so there are always two distinct roots. Go to Quadratics Menu Go to Main Menu EXIT

182 Question 4 Let px2 + 5x – 2p = ax2 + bx + c
The equation of a parabola is f(x) = px2 + 5x – 2p . Prove that the equation f(x) = 0 always has two distinct roots. then a = p, b = 5 & c = -2p. So discriminant = b2 – 4ac = 52 – (4 X p X (-2p)) = 25 – (-8p2) = 8p Since p2  0 for all values of p then 8p > 0. Begin Solution The discriminant is always positive so there are always two distinct roots. Continue Solution Markers Comments Quadratics Menu Back to Home

183 Learn Rules relating to discriminant b2- 4ac b2- 4ac = 0 Equal roots
Markers Comments Learn Rules relating to discriminant b2- 4ac b2- 4ac = Equal roots b2- 4ac > Real, distinct roots b2- 4ac < No real roots b2- 4ac Real roots, equal or distinct Let px2 + 5x – 2p = ax2 + bx + c then a = p, b = 5 & c = -2p. So discriminant = b2 – 4ac = 52 – (4 X p X (-2p)) = 25 – (-8p2) = 8p Since p2  0 for all values of p then 8p > 0. Next Comment The discriminant is always positive so there are always two distinct roots. Quadratics Menu Back to Home

184 State condition you require to show true explicitly:
Markers Comments State condition you require to show true explicitly: For two distinct roots b2- 4ac > 0 Let px2 + 5x – 2p = ax2 + bx + c then a = p, b = 5 & c = -2p. So discriminant = b2 – 4ac = 52 – (4 X p X (-2p)) = 25 – (-8p2) = 8p Since p2  0 for all values of p then 8p > 0. Next Comment The discriminant is always positive so there are always two distinct roots. Quadratics Menu Back to Home

185 algebraic logic or show the graph of 8p2 + 25 is always
Markers Comments To show 8p2 + 25> 0 use algebraic logic or show the graph of 8p is always above the “x-axis”. Let px2 + 5x – 2p = ax2 + bx + c then a = p, b = 5 & c = -2p. So discriminant = b2 – 4ac = 52 – (4 X p X (-2p)) = 25 – (-8p2) 25 = 8p Min. T.P. at (0,25) hence graph always above the “x - axis.” Since p2  0 for all values of p then 8p > 0. Next Comment The discriminant is always positive so there are always two distinct roots. Quadratics Menu Back to Home

186 QUADRATICS : Question 5 Given that the roots of 3x(x + p) = 4p(x – 1) are equal then show that p = 0 or p = 48. Reveal answer only Go to full solution Go to Marker’s Comments Go to Quadratics Menu Go to Main Menu EXIT

187 QUADRATICS : Question 5 Given that the roots of 3x(x + p) = 4p(x – 1) are equal then show that p = 0 or p = 48. For equal roots we need discriminant = 0 p(p ) = 0 Reveal answer only ie p = 0 or p = 48 Go to full solution Go to Marker’s Comments Go to Quadratics Menu Go to Main Menu EXIT

188 Question 5 Rearranging 3x(x + p) = 4p(x – 1) 3x2 + 3px = 4px - 4p
Given that the roots of 3x(x + p) = 4p(x – 1) are equal then show that p = 0 or p = 48. 3x2 + 3px = 4px - 4p 3x2 - px + 4p = 0 Let 3x2 - px + 4p = ax2 + bx + c then a = 3, b = -p & c = 4p For equal roots we need discriminant = 0 ie b2 – 4ac = 0 (-p)2 - (4 X 3 X 4p) = 0 p p = 0 Begin Solution p(p ) = 0 Continue Solution ie p = 0 or p = 48 Markers Comments Quadratics Menu Back to Home

189 Must put the equation into standard quadratic form before
Markers Comments Must put the equation into standard quadratic form before reading off a,b and c. i.e. ax2 + bx +c = 0 Rearranging 3x(x + p) = 4p(x – 1) 3x2 + 3px = 4px - 4p 3x2 - px + 4p = 0 Let 3x2 - px + 4p = ax2 + bx + c then a = 3, b = -p & c = 4p For equal roots we need discriminant = 0 ie b2 – 4ac = 0 (-p)2 - (4 X 3 X 4p) = 0 p p = 0 Next Comment p(p ) = 0 Quadratics Menu ie p = 0 or p = 48 Back to Home

190 Learn Rules relating to discriminant b2- 4ac b2- 4ac = 0 Equal roots
Markers Comments Learn Rules relating to discriminant b2- 4ac b2- 4ac = Equal roots b2- 4ac > Real, distinct roots b2- 4ac < No real roots b2- 4ac Real roots, equal or distinct Rearranging 3x(x + p) = 4p(x – 1) 3x2 + 3px = 4px - 4p 3x2 - px + 4p = 0 Let 3x2 - px + 4p = ax2 + bx + c then a = 3, b = -p & c = 4p For equal roots we need discriminant = 0 ie b2 – 4ac = 0 (-p)2 - (4 X 3 X 4p) = 0 p p = 0 Next Comment p(p ) = 0 Quadratics Menu ie p = 0 or p = 48 Back to Home

191 State condition you require explicitly:
Markers Comments State condition you require explicitly: For two equal roots b2- 4ac = 0 Rearranging 3x(x + p) = 4p(x – 1) 3x2 + 3px = 4px - 4p 3x2 - px + 4p = 0 Let 3x2 - px + 4p = ax2 + bx + c Must use factorisation to solve resulting quadratic. then a = 3, b = -p & c = 4p For equal roots we need discriminant = 0 ie b2 – 4ac = 0 (-p)2 - (4 X 3 X 4p) = 0 p p = 0 Next Comment p(p ) = 0 Quadratics Menu ie p = 0 or p = 48 Back to Home

192 QUADRATICS : Question 6 The diagram below shows the parabola y = -2x2 + 3x + 2 and the line x + y – 4 = 0. y = -2x2 + 3x + 2 x + y – 4 = 0 Reveal answer only Go to full solution Go to Marker’s Comms Go to Quadratics Menu Go to Main Menu Prove that the line is a tangent to the curve. EXIT

193 QUADRATICS : Question 6 The diagram below shows the parabola y = -2x2 + 3x + 2 and the line x + y – 4 = 0. y = -2x2 + 3x + 2 x + y – 4 = 0 Reveal answer only Go to full solution Go to Marker’s Comms Go to Quadratics Menu Go to Main Menu Since the discriminant = 0 then there is only one solution to the equation so only one point of contact and it follows that the line is a tangent. Prove that the line is a tangent to the curve. EXIT

194 Question 6 Linear equation can be changed from
x + y – 4 = 0 to y = -x + 4. The line and curve meet when y = -x + 4 and y = -2x2 + 3x + 2 . x + y – 4 = 0 So -x = -2x2 + 3x + 2 Or 2x2 - 4x + 2 = 0 Let 2x2 - 4x + 2 = ax2 + bx + c y = -2x2 + 3x + 2 Prove that the line is a tangent. then a = 2, b = -4 & c = 2. Begin Solution Continue Solution Markers Comments Quadratics Menu Back to Home

195 Question 6 then a = 2, b = -4 & c = 2. So discriminant = b2 – 4ac
= (-4)2 – (4 X 2 X 2) x + y – 4 = 0 = = 0 Since the discriminant = 0 then there is only one solution to the equation so only one point of contact and it follows that the line is a tangent. y = -2x2 + 3x + 2 Prove that the line is a tangent. Begin Solution Continue Solution Markers Comments Quadratics Menu Back to Home

196 For intersection of line and polynomial y1 = y2
Markers Comments For intersection of line and polynomial y1 = y2 Terms to the left, simplify and factorise. Linear equation can be changed from x + y – 4 = 0 to y = -x + 4. The line and curve meet when y = -x + 4 and y = -2x2 + 3x + 2 . So -x = -2x2 + 3x + 2 Or 2x2 - 4x + 2 = 0 Let 2x2 - 4x + 2 = ax2 + bx + c then a = 2, b = -4 & c = 2. Next Comment Quadratics Menu Back to Home

197 “equal roots” may be used in place of the discriminant.
Markers Comments Or x + 4 = -2x2 + 3x + 2 2x2 - 4x + 2 = 0 2(x2 - 2x + 1) = 0 2(x - 1)(x - 1) = 0 x = 1 (twice) Equal roots tangency then a = 2, b = -4 & c = 2. So discriminant = b2 – 4ac = (-4)2 – (4 X 2 X 2) = = 0 To prove tangency “equal roots” may be used in place of the discriminant. The statement must be made explicitly. Since the discriminant = 0 then there is only one solution to the equation so only one point of contact and it follows that the line is a tangent. Next Comment Quadratics Menu Back to Home

198 HIGHER – ADDITIONAL QUESTION BANK
You have chosen to study: UNIT 2 : Integration Please choose a question to attempt from the following: 1 2 3 4 5 Back to Unit 2 Menu EXIT

199 INTEGRATION : Question 1
The diagram below shows the curve y = x2 - 8x + 18 and the lines x = 3 and x = k. y = x2 - 8x + 18 x = x = k Reveal answer only Go to full solution Go to Marker’s Comments Show that the shaded area is given by 1/3k3 – 4k2 + 18k - 27 Go to Integration Menu Go to Main Menu EXIT

200  INTEGRATION : Question 1
The diagram below shows the curve y = x2 - 8x + 18 and the lines x = 3 and x = k. y = x2 - 8x + 18 x = x = k Reveal answer only Go to full solution Go to Marker’s Comments Show that the shaded area is given by 1/3k3 – 4k2 + 18k - 27 Go to Integration Menu Go to Main Menu Area = (x2 - 8x + 18) dx 3 k EXIT = 1/3k3 – 4k2 + 18k – 27 as required.

201 [ ] [ ]  Question 1 = x3 - 8x2 + 18x Area = (x2 - 8x + 18) dx
k The diagram shows the curve y = x2 - 8x + 18 and the lines x = 3 and x = k. = x3 - 8x2 + 18x [ ] k 3 = 1/3x3 – 4x2 + 18x [ ] k 3 Show that the shaded area is given by 1/3k3 – 4k2 + 18k - 27 = (1/3k3 – 4k2 + 18k) – ((1/3 X 27) – (4 X 9) + 54) = 1/3k3 – 4k2 + 18k – 27 as required. Begin Solution Continue Solution Markers Comments Integration Menu Back to Home

202 [ ] [ ]  Learn result can be used to find the enclosed area shown:
Markers Comments Learn result can be used to find the enclosed area shown: Area = (x2 - 8x + 18) dx 3 k = x3 - 8x2 + 18x [ ] k 3 = 1/3x3 – 4x2 + 18x [ ] k 3 a b f(x) = (1/3k3 – 4k2 + 18k) – ((1/3 X 27) – (4 X 9) + 54) = 1/3k3 – 4k2 + 18k – 27 as required. Next Comment Integration Menu Back to Home

203 [ ] [ ]  Learn result for integration: “Add 1 to the power and
Markers Comments Learn result for integration: “Add 1 to the power and divide by the new power.” Area = (x2 - 8x + 18) dx 3 k = x3 - 8x2 + 18x [ ] k 3 = 1/3x3 – 4x2 + 18x [ ] k 3 = (1/3k3 – 4k2 + 18k) – ((1/3 X 27) – (4 X 9) + 54) = 1/3k3 – 4k2 + 18k – 27 as required. Next Comment Integration Menu Back to Home

204 INTEGRATION : Question 2
Given that dy/dx = 12x2 – 6x and the curve y = f(x) passes through the point (2,15) then find the equation of the curve y = f(x). Reveal answer only Go to full solution Go to Marker’s Comments Go to Integration Menu Go to Main Menu EXIT

205 INTEGRATION : Question 2
Given that dy/dx = 12x2 – 6x and the curve y = f(x) passes through the point (2,15) then find the equation of the curve y = f(x). Equation of curve is y = 4x3 – 3x2 - 5 Reveal answer only Go to full solution Go to Marker’s Comments Go to Integration Menu Go to Main Menu EXIT

206 Question 2 dy/dx = 12x2 – 6x Given that dy/dx = 12x2 – 6x and
the curve y = f(x) passes through the point (2,15) then find the equation of the curve y = f(x). So = 12x3 – 6x2 + C = 4x3 – 3x2 + C Substituting (2,15) into y = 4x3 – 3x2 + C We get = (4 X 8) – (3 X 4) + C So C = 15 ie C = -5 Begin Solution Equation of curve is y = 4x3 – 3x2 - 5 Continue Solution Markers Comments Integration Menu Back to Home

207 Learn the result that integration undoes differentiation: i.e. given
Markers Comments Learn the result that integration undoes differentiation: i.e. given dy/dx = 12x2 – 6x So = 12x3 – 6x2 + C = 4x3 – 3x2 + C Learn result for integration: “Add 1 to the power and divide by the new power”. Substituting (2,15) into y = 4x3 – 3x2 + C We get = (4 X 8) – (3 X 4) + C So C = 15 ie C = -5 Equation of curve is y = 4x3 – 3x2 - 5 Next Comment Integration Menu Back to Home

208 Do not forget the constant of integration!!!
Markers Comments Do not forget the constant of integration!!! dy/dx = 12x2 – 6x So = 12x3 – 6x2 + C = 4x3 – 3x2 + C Substituting (2,15) into y = 4x3 – 3x2 + C We get = (4 X 8) – (3 X 4) + C So C = 15 ie C = -5 Equation of curve is y = 4x3 – 3x2 - 5 Next Comment Integration Menu Back to Home

209  INTEGRATION : Question 3 Find x2 - 4 dx 2xx Reveal answer only
Go to full solution Go to Marker’s Comments Go to Integration Menu Go to Main Menu EXIT

210  INTEGRATION : Question 3 Find x2 - 4 dx 2xx = xx + 4 + C 3 x
Reveal answer only Go to full solution Go to Marker’s Comments Go to Integration Menu Go to Main Menu EXIT

211     Question 3 x2 - 4 dx 2xx Find x2 - 4 dx 2xx = x2 - 4 dx
= /3 X 1/2x3/ (-2) X 2x-1/ C = /3x3/ x-1/ C = xx C x Begin Solution Continue Solution Markers Comments Integration Menu Back to Home

212    Prepare expression by: 1 Dividing out the fraction.
Markers Comments Prepare expression by: 1 Dividing out the fraction. 2 Applying the laws of indices. x 2xx dx = x 2x3/ x3/2 dx Learn result for integration: Add 1 to the power and divide by the new power. = 1/2x1/ x-3/2 dx = /3 X 1/2x3/ (-2) X 2x-1/ C = /3x3/ x-1/ C Do not forget the constant of integration. = xx C x Next Comment Integration Menu Back to Home

213  ( ) INTEGRATION : Question 4 Evaluate x2 - 2 2 dx x
1 2  ( ) Evaluate x dx x Reveal answer only Go to full solution Go to Marker’s Comments Go to Integration Menu Go to Main Menu EXIT

214  ( ) INTEGRATION : Question 4 = 21/5 Evaluate x2 - 2 2 dx x
 ( ) Evaluate x dx x = 21/5 Reveal answer only Go to full solution Go to Marker’s Comments Go to Integration Menu Go to Main Menu EXIT

215  ( )  ( )   [ ] [ ] ( ) ( ) Question 4 = 21/5 x2 - 2 2 dx x
 ( ) x dx x 1 2  ( ) Evaluate x dx x ( ) = x x dx x2 1 2 ( ) = x x + 4x-2 dx 1 2 [ ] = x5 - 4x x-1 1 2 = x x x [ ] 2 1 = (32/ ) - (1/ ) Begin Solution = 21/5 Continue Solution Markers Comments Integration Menu Back to Home

216  ( )   [ ] [ ] ( ) ( ) Prepare expression by:
Markers Comments Prepare expression by: 1 Expanding the bracket 2 Applying the laws of indices. 1 2  ( ) x dx x ( ) = x x dx x2 1 2 Learn result for integration: “Add 1 to the power and divide by the new power”. ( ) = x x + 4x-2 dx 1 2 [ ] = x5 - 4x x-1 1 2 = x x x [ ] 2 1 When applying limits show substitution clearly. = (32/ ) - (1/ ) = 21/5 Next Comment Integration Menu Back to Home

217 INTEGRATION : Question 5
The diagram below shows the parabola y = -x2 + 8x - 10 and the line y = x. They meet at the points A and B. y = -x2 + 8x - 10 y = x A B Reveal answer only Go to full solution Go to Marker’s Comms Go to Integration Menu Go to Main Menu (a) Find the coordinates of A and B. Hence find the shaded area between the curves. EXIT

218 INTEGRATION : Question 5
The diagram below shows the parabola y = -x2 + 8x - 10 and the line y = x. They meet at the points A and B. y = -x2 + 8x - 10 y = x A B Reveal answer only Go to full solution Go to Marker’s Comms Go to Integration Menu Go to Main Menu (a) Find the coordinates of A and B. A is (2,2) and B is (5,5) . Hence find the shaded area between the curves. = 41/2units2 EXIT

219 Question 5 Line & curve meet when y = x and y = -x2 + 8x - 10 .
The diagram shows the parabola y = -x2 + 8x - 10 and the line y = x. They meet at the points A and B. So x = -x2 + 8x - 10 or x2 - 7x + 10 = 0 (a) Find the coordinates of A and B. ie (x – 2)(x – 5) = 0 Hence find the shaded area between the curves. ie x = 2 or x = 5 Since points lie on y = x then A is (2,2) and B is (5,5) . Begin Solution Continue Solution Markers Comments Integration Menu Back to Home

220   [ ] Question 5 A is (2,2) and B is (5,5) .
The diagram shows the parabola y = -x2 + 8x - 10 and the line y = x. They meet at the points A and B. (b) Curve is above line between limits so Shaded area = (-x2 + 8x – x) dx 2 5 = (-x2 + 7x – 10) dx 2 5 (a) Find the coordinates of A and B. = x x x [ ] 5 2 Hence find the shaded area between the curves. = (-125/ /2 – 50) – (-8/3 +14 – 20) Begin Solution = 41/2units2 Continue Solution Markers Comments Integration Menu Back to Home

221 At intersection of line and curve y1 = y2
Markers Comments At intersection of line and curve y1 = y2 Terms to the left, simplify and factorise. Line & curve meet when y = x and y = -x2 + 8x So x = -x2 + 8x - 10 or x2 - 7x + 10 = 0 ie (x – 2)(x – 5) = 0 ie x = 2 or x = 5 Since points lie on y = x then A is (2,2) and B is (5,5) . Next Comment Integration Menu Back to Home

222   [ ] Learn result can be used to find the enclosed area shown: y1
Markers Comments Learn result can be used to find the enclosed area shown: (b) Curve is above line between limits so Shaded area = (-x2 + 8x – x) dx 2 5 = (-x2 + 7x – 10) dx 2 5 = x x x [ ] 5 2 a b upper curve y1 area y2 lower curve = (-125/ /2 – 50) – (-8/3 +14 – 20) = 41/2units2 Next Comment Integration Menu Back to Home

223 HIGHER – ADDITIONAL QUESTION BANK
You have chosen to study: UNIT 2 : Addition Formulae Please choose a question to attempt from the following: 1 2 3 4 Back to Unit 2 Menu EXIT

224 ADDITION FORMULAE : Question 1
In triangle PQR show that the exact value of cos(a - b) is 4/5. P Q R 1 4 2 a b Reveal answer only Go to full solution Go to Marker’s Comments Go to Add Formulae Menu Go to Main Menu EXIT

225 ADDITION FORMULAE : Question 1
In triangle PQR show that the exact value of cos(a - b) is 4/5. P Q R 1 4 2 a b cos(a – b) = cosacosb + sinasinb = (2/5 X 1/5 ) + (1/5 X 2/5 ) = 4/5 Reveal answer only Go to full solution Go to Marker’s Comments Go to Add Formulae Menu Go to Main Menu EXIT

226 Question 1 PQ2 = 12 + 22 = 5 In triangle PQR show that the
exact value of cos(a - b) is 4/5. so PQ = 5 QR2 = = 20 so QR = 20 = 45 = 25 P Q R 1 4 2 a b sina = 2/25 = 1/5 & sinb = 2/5 cosa = 4/25 = 2/5 & cosb = 1/5 cos(a – b) = cosacosb + sinasinb = (2/5 X 1/5 ) + (1/5 X 2/5 ) Begin Solution = 2/ /5 Continue Solution = 4/5 Markers Comments Add Formulae Menu Back to Home

227 Use formula sheet to check correct expansion and relate
Markers Comments Use formula sheet to check correct expansion and relate to given variables: cos(a - b) = cosacosb + sina sinb PQ2 = = 5 so PQ = 5 QR2 = = 20 so QR = 20 = 45 = 25 sina = 2/25 = 1/5 & sinb = 2/5 Work only with exact values when applying Pythagoras’: cosa = 4/25 = 2/5 & cosb = 1/5 2 a a = cos(a – b) = cosacosb + sinasinb = (2/5 X 1/5 ) + (1/5 X 2/5 ) = 2/ /5 Next Comment Add Form Menu = 4/5 Back to Home

228 ADDITION FORMULAE : Question 2
Find the exact value of cos(p + q) in the diagram below Y X O F(4,4) G(3,-1) p q Reveal answer only Go to full solution Go to Marker’s Comments Go to Add Formulae Menu Go to Main Menu EXIT

229 ADDITION FORMULAE : Question 2
Find the exact value of cos(p + q) in the diagram below Y X O F(4,4) G(3,-1) p q Reveal answer only = 1/5 Go to full solution Go to Marker’s Comments Go to Add Formulae Menu Go to Main Menu EXIT

230 Question 2 OF2 = 42 + 42 = 32 Find the exact value of cos(p + q)
in the diagram below so OF = 32 = 162 = 42 OG2 = = 10 so OG = 10 Y X O F(4,4) G(3,-1) p q sinp = 4/42 = 1/2 & sinq = 1/10 cosp = 4/42 = 1/2 & cosq = 3/10 cos(p + q) = cospcosq - sinpsinq = (1/2 X 3/10 ) - (1/2 X 1/10 ) = 3/ /20 Begin Solution = 2/20 Continue Solution = 2/4 5 Markers Comments = 2/2 5 Add Formulae Menu = 1/5 Back to Home

231 Use formula sheet to check correct expansion and relate
Markers Comments Use formula sheet to check correct expansion and relate to given variables: cos(a + b) = cosacosb - sina sinb cos(p + q) = cospcosq - sinpsinq OF2 = = 32 so OF = 32 = 162 = 42 OG2 = = 10 Formula Sheet: so OG = 10 sinp = 4/42 = 1/2 & sinq = 1/10 becomes: cosp = 4/42 = 1/2 & cosq = 3/10 cos(p + q) = cospcosq - sinpsinq = (1/2 X 3/10 ) - (1/2 X 1/10 ) = 3/ /20 = 2/20 = 2/4 5 Next Comment = 2/2 5 Add Form Menu = 1/5 Back to Home

232 Work only with exact values when applying Pythagoras’:
Markers Comments Work only with exact values when applying Pythagoras’: OF2 = = 32 so OF = 32 = 162 = 42 OG2 = = 10 2 a a = so OG = 10 sinp = 4/42 = 1/2 & sinq = 1/10 cosp = 4/42 = 1/2 & cosq = 3/10 cos(p + q) = cospcosq - sinpsinq = (1/2 X 3/10 ) - (1/2 X 1/10 ) = 3/ /20 = 2/20 = 2/4 5 Next Comment = 2/2 5 Add Form Menu = 1/5 Back to Home

233 ADDITION FORMULAE : Question 3
Solve the equation sinx° = 3cos2x° where 0<x<360. Reveal answer only Go to full solution Go to Marker’s Comments Go to Add Formulae Menu Go to Main Menu EXIT

234 ADDITION FORMULAE : Question 3
Solve the equation sinx° = 3cos2x° where 0<x<360. Solution = {30, 150, 199.5, 340.5} Reveal answer only Go to full solution Go to Marker’s Comments Go to Add Formulae Menu Go to Main Menu EXIT

235 Question 3 2 - sinx° = 3cos2x° 2 - sinx° = 3(1 – 2sin2x°)
Solve the equation 2 - sinx° = 3cos2x° where 0<x<360. 2 - sinx° = 3(1 – 2sin2x°) 2 - sinx° = 3 – 6sin2x° Rearrange into quadratic form 6sin2x° - sinx° - 1 = 0 6s2 – s – 1 = (3s + 1)(2s – 1) (3sinx° + 1)(2sinx° - 1) = 0 sinx° = -1/3 or sinx° = 1/2 Q3 or Q4 Q1 or Q2 Begin Solution A S T C (180 - a)° (180 + a)° (360 - a)° Continue Solution Markers Comments Add Formulae Menu Back to Home

236 Question 3 sinx° = -1/3 or sinx° = 1/2 Solve the equation
2 - sinx° = 3cos2x° where 0<x<360. A S T C (180 - a)° (180 + a)° (360 - a)° sin-1 (1/2) = 30° Q1: x = 30 Q2: x = 180 – 30 = 150 sin-1 (1/3) = 19.5° Begin Solution Q3: x = = 199.5 Q4: x = = 340.5 Continue Solution Markers Comments Solution = {30, 150, 199.5, 340.5} Add Formulae Menu Back to Home

237 Use formula sheet to check correct expansion and relate to
Markers Comments Use formula sheet to check correct expansion and relate to given variables: cos2x° =1 - 2sin2x° 2 - sinx° = 3cos2x° 2 - sinx° = 3(1 – 2sin2x°) 2 - sinx° = 3 – 6sin2x° Rearrange into quadratic form 6sin2x° - sinx° - 1 = 0 Formula must be consistent with the rest of the equation. 2 - sinx° i.e. choose formula with sinx° 6s2 – s – 1 = (3s + 1)(2s – 1) (3sinx° + 1)(2sinx° - 1) = 0 sinx° = -1/3 or sinx° = 1/2 Q3 or Q4 Q1 or Q2 A S T C (180 - a)° (180 + a)° (360 - a)° Next Comment Add Form Menu Back to Home

238 Only one way to solve the resulting quadratic:
Markers Comments Only one way to solve the resulting quadratic: Terms to the left, put in standard quadratic form and factorise. 2 - sinx° = 3cos2x° 2 - sinx° = 3(1 – 2sin2x°) 2 - sinx° = 3 – 6sin2x° Rearrange into quadratic form 6sin2x° - sinx° - 1 = 0 6s2 – s – 1 = (3s + 1)(2s – 1) (3sinx° + 1)(2sinx° - 1) = 0 sinx° = -1/3 or sinx° = 1/2 Q3 or Q4 Q1 or Q2 A S T C (180 - a)° (180 + a)° (360 - a)° Next Comment Add Form Menu Back to Home

239 Take care to relate “solutions” to the given domain.
Markers Comments Take care to relate “solutions” to the given domain. Since all 4 values are included. sinx° = -1/3 or sinx° = 1/2 A S T C (180 - a)° (180 + a)° (360 - a)° sin-1 (1/2) = 30° Q1: x = 30 Q2: x = 180 – 30 = 150 sin-1 (1/3) = 19.5° Q3: x = = 199.5 Q4: x = = 340.5 Next Comment Add Form Menu Solution = {30, 150, 199.5, 340.5} Back to Home

240 ADDITION FORMULAE : Question 4
Solve sin2 = cos  where 0 <  < 2 Reveal answer only Go to full solution Go to Marker’s Comments Go to Add Formulae Menu Go to Main Menu EXIT

241 ADDITION FORMULAE : Question 4
Solve sin2 = cos  where 0 <  < 2 Soltn = {/6 , /2 , 5/6 , 3/2} Reveal answer only Go to full solution Go to Marker’s Comments Go to Add Formulae Menu Go to Main Menu EXIT

242 Question 4 sin2 = cos sin2 - cos = 0 Solve sin2 = cos 
where 0 <  < 2 2sin cos - cos = 0 (common factor cos) cos(2sin - 1) = 0 cos = 0 or (2sin - 1) = 0 cos = 0 or sin = 1/2 (roller-coaster graph) Q1 or Q2  = /2 or 3/2 Begin Solution  -    +  2 -  A S T C Continue Solution Markers Comments Add Formulae Menu Back to Home

243 Question 4 cos = 0 or sin = 1/2 Solve sin2 = cos   = /2 or 3/2
where 0 <  < 2  = /2 or 3/2 Q1 or Q2  -    +  2 -  A S T C sin-1(1/2) = /6 Q1:  = /6 Q2:  =  - /6 = 5/6 Begin Solution Continue Solution Markers Comments Soltn = {/6 , /2 , 5/6 , 3/2} Add Formulae Menu Back to Home

244 Use formula sheet to check correct expansion and relate to
Markers Comments Use formula sheet to check correct expansion and relate to given variables: sin2x = 2sinxcosx sin2 = cos sin2 - cos = 0 2sin cos - cos = 0 (common factor cos) Although equation is in radians possible to work in degrees and convert to radians using: cos(2sin - 1) = 0 cos = 0 or (2sin - 1) = 0 cos = 0 or sin = 1/2 (roller-coaster graph) Q1 or Q2  = /2 or 3/2  -    +  2 -  A S T C Next Comment Add Form Menu Back to Home

245 Only one way to solve the result:
Markers Comments Only one way to solve the result: Terms to the left, put in standard quadratic form and factorise. sin2 = cos sin2 - cos = 0 2sin cos - cos = 0 (common factor cos) cos(2sin - 1) = 0 cos = 0 or (2sin - 1) = 0 cos = 0 or sin = 1/2 (roller-coaster graph) Q1 or Q2  = /2 or 3/2  -    +  2 -  A S T C Next Comment Add Form Menu Back to Home

246 use sketch of graph to obtain solutions: cosx = 0
Markers Comments For trig. equations sinx = 0 or 1 or cosx = 0 or 1 use sketch of graph to obtain solutions: cosx = 0 cos = 0 or sin = 1/2  = /2 or 3/2 Q1 or Q2  -    +  2 -  A S T C y x y = cos x x = , sin-1(1/2) = /6 Q1:  = /6 Q2:  =  - /6 = 5/6 Next Comment Add Form Menu Soltn = {/6 , /2 , 5/6 , 3/2} Back to Home

247 Take care to relate “solutions” to the given domain.
Markers Comments Take care to relate “solutions” to the given domain. Since all 4 values are included cos = 0 or sin = 1/2  = /2 or 3/2 Q1 or Q2  -    +  2 -  A S T C sin-1(1/2) = /6 Q1:  = /6 Q2:  =  - /6 = 5/6 Next Comment Add Form Menu Soltn = {/6 , /2 , 5/6 , 3/2} Back to Home

248 HIGHER – ADDITIONAL QUESTION BANK
You have chosen to study: UNIT 2 : The Circle Please choose a question to attempt from the following: 1 2 3 4 Back to Unit 2 Menu EXIT

249 THE CIRCLE : Question 1 The face of a stopwatch can be modelled by the circle with equation x2 + y2 – 10x – 8y + 5 = 0. The centre is at C and the winder is at W. The dial for the second hand is 1/3 the size of the face and is located half way between C and W. W C Find the coordinates of C and W. (b) Hence find the equation of the dial for the second hand. Reveal answer only Go to Marker’s Comments EXIT Go to full solution Go to Circle Menu

250 THE CIRCLE : Question 1 The face of a stopwatch can be modelled by the circle with equation x2 + y2 – 10x – 8y + 5 = 0. The centre is at C and the winder is at W. The dial for the second hand is 1/3 the size of the face and is located half way between C and W. W C C is (5,4) W is (5,10). Find the coordinates of C and W. (b) Hence find the equation of the dial for the second hand. (x – 5)2 + (y – 7)2 = 4 Reveal answer only Go to Marker’s Comments EXIT Go to full solution Go to Circle Menu

251 Question 1 Find the coordinates of C and W.
The face of a stopwatch can be modelled by the circle with equation x2 + y2 – 10x – 8y + 5 = 0. The centre is at C and the winder is at W. The dial for the second hand is 1/3 the size of the face and is located half way between C and W. Large circle is x2 + y2 – 10x – 8y + 5 = 0. Comparing Coefficients x2 + y2 + 2gx + 2fy + c = 0 2g = -10, 2f = -8 and c = 5 So g = -5, f = -4 and c = 5 Centre is (-g,-f) and r = (g2 + f2 – c) C is (5,4) r = ( – 5) r = 36 = 6 Begin Solution W is 6 units above C so W is (5,10). Continue Solution Markers Comments Circle Menu Back to Home

252 Question 1 (b) Hence find the equation of the dial for the second hand. The face of a stopwatch can be modelled by the circle with equation x2 + y2 – 10x – 8y + 5 = 0. The centre is at C and the winder is at W. The dial for the second hand is 1/3 the size of the face and is located half way between C and W. Radius of small circle = 6  3 = 2. Centre is midpoint of CW ie (5,7). Using (x – a)2 + (y – b)2 = r2 we get (x – 5)2 + (y – 7)2 = 4 Begin Solution Continue Solution Markers Comments Circle Menu Back to Home

253 Use formula sheet to check correct formulas and relate to
Markers Comments Use formula sheet to check correct formulas and relate to general equation of the circle. Find the coordinates of C and W. Large circle is x2 + y2 – 10x – 8y + 5 = 0. Comparing Coefficients x2 + y2 + 2gx + 2fy + c = 0 2g = -10, 2f = -8 and c = 5 So g = -5, f = -4 and c = 5 Centre is (-g,-f) and r = (g2 + f2 – c) C is (5,4) r = ( – 5) r = 36 = 6 Next Comment W is 6 units above C so W is (5,10). Circle Menu Back to Home

254 Identify centre and radius before putting values into circle equation:
Markers Comments Identify centre and radius before putting values into circle equation: Centre (5,7), radius = 2 (x - a)2 + (y - b)2= r2 (x - 5)2 + (y - 7)2= 22 There is no need to expand this form of the circle equation. (b) Hence find the equation of the dial for the second hand. Radius of small circle = 6  3 = 2. Centre is midpoint of CW ie (5,7). Using (x – a)2 + (y – b)2 = r2 we get (x – 5)2 + (y – 7)2 = 4 Next Comment Circle Menu Back to Home

255 THE CIRCLE : Question 2 In a factory a conveyor belt passes through several rollers. Part of this mechanism is shown below. (a) The small roller has centre P and equation x2 + y2 – 6x + 2y - 7 = 0. belt P A(7,0) B Q The belt is a common tangent which meets the small roller at A(7,0)and the large roller at B. Find the equation of the tangent. B has an x-coordinate of 10. Find its y-coordinate. (b) Find the equation of the larger roller given that its diameter is twice that of the smaller roller, and has centre Q. Reveal answer only Go to Marker’s Comments EXIT Go to full solution Go to Circle Menu

256 THE CIRCLE : Question 2 In a factory a conveyor belt passes through several rollers. Part of this mechanism is shown below. (a) The small roller has centre P and equation x2 + y2 – 6x + 2y - 7 = 0. belt P A(7,0) B Q The belt is a common tangent which meets the small roller at A(7,0)and the large roller at B. Find the equation of the tangent. B has an x-coordinate of 10. Find its y-coordinate. y = -4x + 28 B is (10,-12) (b) Find the equation of the larger roller given that its diameter is twice that of the smaller roller, and has centre Q. (x – 18)2 + (y + 10)2 = 68 Reveal answer only Go to Marker’s Comments EXIT Go to full solution Go to Circle Menu

257 Question 2 = 1/4 (a) (i) Small circle is x2 + y2 – 6x + 2y - 7 = 0.
The small roller has centre P and equation x2 + y2 – 6x + 2y - 7 = 0. Comparing coefficients x2 + y2 + 2gx + 2fy + c = 0 The belt is a common tangent which meets the small roller at A(7,0)and the large roller at B. Find the equation of the tangent. B has an x-coordinate of 10. Find its y-coordinate. 2g = -6, 2f = 2 and c = -7 So g = -3, f = 1 and c = -7 Centre is (-g,-f) and r = (g2 + f2 – c) P is (3,-1) r = ( ) r = 17 Begin Solution For equation of tangent: Continue Solution Gradient of PA = 0 – (-1) 7 - 3 = 1/4 Markers Comments Circle Menu Back to Home

258 Question 2 = 1/4 P is (3,-1) The small roller has centre P
and equation x2 + y2 – 6x + 2y - 7 = 0. For equation of tangent: Gradient of PA = 0 – (-1) 7 - 3 = 1/4 The belt is a common tangent which meets the small roller at A(7,0)and the large roller at B. Find the equation of the tangent. B has an x-coordinate of 10. Find its y-coordinate. Gradient of tangent = -4 (as m1m2 = -1) Using y – b = m(x – a) with (a,b) = (7,0) & m = -4 we get …. y – 0 = -4(x – 7) or y = -4x + 28 Begin Solution Continue Solution Markers Comments Circle Menu Back to Home

259 Question 2 (a)(ii) At B x = 10 so y = (-4 X 10) + 28 = -12.
The small roller has centre P and equation x2 + y2 – 6x + 2y - 7 = 0. ie B is (10,-12) The belt is a common tangent which meets the small roller at A(7,0)and the large roller at B. Find the equation of the tangent. B has an x-coordinate of 10. Find its y-coordinate. Begin Solution Continue Solution Markers Comments Circle Menu Back to Home

260 Question 2 r = 17 P is (3,-1) The small roller has centre P
Small Circle: r = 17 P is (3,-1) A(7,0) The small roller has centre P and equation x2 + y2 – 6x + 2y - 7 = 0. (b) From P to A is 4 along and 1 up. So from B to Q is 8 along and 2 up. Q is at (10+8, -12+2) ie (18,-10) (b) Find the equation of the larger roller given that its diameter is twice that of the smaller roller, and has centre Q. Radius of larger circle is 217 so r2 = (217)2 = 4 X 17 = 68 Using (x – a)2 + (y – b)2 = r2 we get (x – 18)2 + (y + 10)2 = 68 Begin Solution Continue Solution Markers Comments Circle Menu Back to Home

261 Use formula sheet to check correct formulas and relate to
Markers Comments Use formula sheet to check correct formulas and relate to general equation of the circle. (a) (i) Small circle is x2 + y2 – 6x + 2y - 7 = 0. Comparing coefficients x2 + y2 + 2gx + 2fy + c = 0 2g = -6, 2f = 2 and c = -7 So g = -3, f = 1 and c = -7 Centre is (-g,-f) and r = (g2 + f2 – c) P is (3,-1) r = ( ) r = 17 For equation of tangent: Next Comment Gradient of PA = 0 – (-1) 7 - 3 = 1/4 Circle Menu Back to Home

262 Identify centre and radius before putting values into circle equation:
Markers Comments Identify centre and radius before putting values into circle equation: Centre (18,-10), radius = 2 (x - a)2 + (y - b)2= r2 (x - 18)2 + (y + 10)2= (2 ) 2 (b) From P to A is 4 along and 1 up. So from B to Q is 8 along and 2 up. Q is at (10+8, -12+2) ie (18,-10) Radius of larger circle is 217 so r2 = (217)2 = 4 X 17 = 68 There is no need to expand this form of the circle equation. If the radius is “squared out” it must be left as an exact value. Using (x – a)2 + (y – b)2 = r2 we get (x – 18)2 + (y + 10)2 = 68 Next Comment Circle Menu Back to Home

263 THE CIRCLE : Question 3 Part of the mechanism inside an alarm clock consists of three different sized collinear cogwheels. C D E The wheels can be represented by circles with centres C, D and E as shown. The small circle with centre C has equation x2 + (y –12)2 = 5. The medium circle with centre E has equation (x - 18)2 + (y –3)2 = 20. Find the equation of the large circle. Reveal answer only Go to Marker’s Comments EXIT Go to full solution Go to Circle Menu

264 THE CIRCLE : Question 3 Part of the mechanism inside an alarm clock consists of three different sized collinear cogwheels. C D E The wheels can be represented by circles with centres C, D and E as shown. The small circle with centre C has equation x2 + (y –12)2 = 5. The medium circle with centre E has equation (x - 18)2 + (y –3)2 = 20. Find the equation of the large circle. (x – 8)2 + (y – 8)2 = 45 Reveal answer only Go to Marker’s Comments EXIT Go to full solution Go to Circle Menu

265 Question 3 Small circle has centre (0,12) and radius 5
The small circle with centre C has equation x2 + (y –12)2 = 5. The medium circle with centre E has equation (x - 18)2 + (y –3)2 = 20. Find the equation of the large circle. Med. circle has centre (18,3) and radius 20 = 4 X 5 = 25 CE2 = (18 – 0)2 + (3 – 12)2 (Distance form!!) = (-9)2 = = 405 CE = 405 = 81 X 5 = 95 Diameter of large circle = 95 - 25 - 5 = 65 Begin Solution C D E 5 35 95 25 Continue Solution Markers Comments So radius of large circle = 35 Circle Menu Back to Home

266 Question 3 = 4/9[( ) - ( )] 25 The small circle with centre C
D E 5 35 95 25 The small circle with centre C has equation x2 + (y –12)2 = 5. The medium circle with centre E has equation (x - 18)2 + (y –3)2 = 20. Find the equation of the large circle. It follows that CD = 4/9CE = 4/9[( ) - ( )] 18 3 12 = 4/9( ) 18 -9 = ( ) 8 -4 So D is the point (0 + 8,12- 4) or (8,8) Using (x – a)2 + (y – b)2 = r2 we get (x – 8)2 + (y – 8)2 = (35)2 Begin Solution or (x – 8)2 + (y – 8)2 = 45 Continue Solution Markers Comments Circle Menu Back to Home

267 Use formula sheet to check correct formulas and relate to
Markers Comments Use formula sheet to check correct formulas and relate to equation of the circle. Small circle has centre (0,12) and radius 5 Med. circle has centre (18,3) and radius 20 = 4 X 5 = 25 CE2 = (18 – 0)2 + (3 – 12)2 (Distance form!!) = (-9)2 = = 405 CE = 405 = 81 X 5 = 95 Diameter of large circle = 95 - 25 - 5 = 65 C D E 5 35 95 25 Next Comment Circle Menu Back to Home

268 The section formula is used to find Centre D
Markers Comments The section formula is used to find Centre D It follows that CD = 4/9CE C E D = 4/9[( ) - ( )] 18 3 12 = 4/9( ) 18 -9 So D is the point (0 + 8,12- 4) or (8,8) Using (x – a)2 + (y – b)2 = r2 we get (x – 8)2 + (y – 8)2 = (35)2 or (x – 8)2 + (y – 8)2 = 45 Next Comment Circle Menu Back to Home

269 THE CIRCLE : Question 4 The central driving wheels on a steam locomotive are linked by a piston rod. A B C E D The rear wheel has centre A & equation x2 + y2 – 4x – 10y + 4 = 0 the front wheel has centre C & equation x2 + y2 – 52x – 10y = 0. If one unit is 5cm then find the size of the minimum gap between the wheels given that the gaps are equal and the wheels are identical. The central wheel has equation x2 + y2 – 28x – 10y = 0 and the midpoint of DE is M(13.5,1.5). Find the equation of DE and hence find the coordinates of D & E. EXIT Go to full solution Reveal answer only

270 THE CIRCLE : Question 4 The central driving wheels on a steam locomotive are linked by a piston rod. The rear wheel has centre A & equation x2 + y2 – 4x – 10y + 4 = 0 the front wheel has centre C & equation x2 + y2 – 52x – 10y = 0. If one unit is 5cm then find the size of the minimum gap between the wheels given that the gaps are equal and the wheels are identical. Each gap = 2 units = 10cm. The central wheel has equation x2 + y2 – 28x – 10y = 0 and the midpoint of DE is M(13.5,1.5). Find the equation of DE and hence find the coordinates of D & E. Hence D is (10,2) and E is (17,1). Equation of DE x + 7y = 24 EXIT Go to full solution Reveal answer only

271 Question 4 (a) Rear wheel is x2 + y2 – 4x – 10y + 4 = 0 .
Comparing coefficients The rear wheel has centre A & equation x2 + y2 – 4x – 10y + 4 = 0 the front wheel has centre C & equation x2 + y2 – 52x – 10y = 0 If one unit is 5cm then find the size of the minimum gap between the wheels. x2 + y2 + 2gx + 2fy + c = 0 2g = -4, 2f = and c = 4 So g = -2, f = and c = 4 Centre is (-g,-f) and r = (g2 + f2 – c) A is (2,5) r = ( – 4) r = 25 = 5 Begin Solution Continue Solution Markers Comments Circle Menu Back to Home

272 Question 4 Front wheel is x2 + y2 – 52x – 10y + 676 = 0
(a) Front wheel is x2 + y2 – 52x – 10y = 0 Comparing coefficients The rear wheel has centre A & equation x2 + y2 – 4x – 10y + 4 = 0 the front wheel has centre C & equation x2 + y2 – 52x – 10y = 0 If one unit is 5cm then find the size of the minimum gap between the wheels. x2 + y2 + 2gx + 2fy + c = 0 2g = -52, 2f = and c = 676 So g = -26, f = -5 and c = 676 Centre is (-g,-f) r = 5 as wheels are identical C is (26,5) AC = 24 units {A is (2,5)} Both gaps = AC – 2 radii - diameter Begin Solution = 24 – 5 – 5 – 10 = 4 units Continue Solution Each gap = 2 units = 10cm. Markers Comments Circle Menu Back to Home

273 Question 4 Middle wheel is x2 + y2 – 28x – 10y + 196 = 0 .
(b) Middle wheel is x2 + y2 – 28x – 10y = 0 . The central wheel has equation x2 + y2 – 28x – 10y = 0 and the midpoint of DE is M(13.5,1.5). Find the equation of DE and hence find the coordinates of D & E. x2 + y2 + 2gx + 2fy + c = 0 2g = -28, 2f = and c = 196 So g = -14, f = and c = 196 Centre is (-g,-f) B is (14,5) Equation of DE Gradient of BM = 5 – 1.5 14 – 13.5 Begin Solution = 3.5/0.5 = 7 Continue Solution Gradient of DE = -1/7 (m1m2 = -1) Markers Comments Circle Menu Back to Home

274 Question 4 Using y – b = m(x – a) The central wheel has equation
Equation of DE Using y – b = m(x – a) The central wheel has equation x2 + y2 – 28x – 10y = 0 and the midpoint of DE is M(13.5,1.5). Find the equation of DE and hence find the coordinates of D & E. we get y – 1.5 = -1/7 (x – 13.5) ( X 7) Or 7y – = -x So DE is x + 7y = 24 x + 7y = 24 can be written as x = 24 – 7y Line & circle meet when x2 + y2 – 28x – 10y = 0 & x = 24 – 7y. Begin Solution Substituting (24 – 7y) for x in the circle equation we get …. Continue Solution Markers Comments Circle Menu Back to Home

275 Question 4 (24 – 7y)2 + y2 – 28(24 – 7y) – 10y + 196 = 0
(b) (24 – 7y)2 + y2 – 28(24 – 7y) – 10y = 0 576 – 336y + 49y2 + y2 – y – 10y = 0 The central wheel has equation x2 + y2 – 28x – 10y = 0 and the midpoint of DE is M(13.5,1.5). Find the equation of DE and Hence find the coordinates of D & E. 50y2 – 150y = 0 (50) y2 – 3y + 2 = 0 (y – 1)(y – 2) = 0 So y = 1 or y = 2 Using x = 24 – 7y If y = 1 then x = 17 If y = 2 then x = 10 Begin Solution Hence D is (10,2) and E is (17,1). Continue Solution Markers Comments Circle Menu Back to Home

276 Use formula sheet to check correct formulas and relate to
Markers Comments Use formula sheet to check correct formulas and relate to general equation of the circle. (a) Rear wheel is x2 + y2 – 4x – 10y + 4 = 0 . Comparing coefficients x2 + y2 + 2gx + 2fy + c = 0 2g = -4, 2f = and c = 4 So g = -2, f = and c = 4 Centre is (-g,-f) and r = (g2 + f2 – c) A is (2,5) r = ( – 4) r = 25 = 5 Next Comment Circle Menu Back to Home

277 In part b) avoid fractions when substituting into the circle equation:
Markers Comments In part b) avoid fractions when substituting into the circle equation: Use x = (24 - 7y) not y = (24 -x) Equation of DE Using y – b = m(x – a) we get y – 1.5 = -1/7 (x – 13.5) Or 7y – = -x So DE is x + 7y = 24 x + 7y = 24 can be written as x = 24 – 7y Line & circle meet when x2 + y2 – 28x – 10y = 0 & x = 24 – 7y. Substituting (24 – 7y) for x in the circle equation we get …. Next Comment Circles Menu Back to Home

278 Even if it appears an error has occurred continue the
Markers Comments (24 – 7y)2 + y2 – 28(24 – 7y) – 10y = 0 576 – 336y + 49y2 + y2 – y – 10y = 0 50y2 – 150y = 0 (50) Even if it appears an error has occurred continue the solution even if it means applying the quadratic formula: y2 – 3y + 2 = 0 (y – 1)(y – 2) = 0 So y = 1 or y = 2 x = Using x = 24 – 7y If y = 1 then x = 17 If y = 2 then x = 10 Hence D is (10,2) and E is (17,1). Next Comment Circle Menu Back to Home

279 Wave Function Vectors Logs & Exponential Further Calculus
HIGHER – ADDITIONAL QUESTION BANK UNIT 3 : Wave Function Vectors Logs & Exponential Further Calculus EXIT

280 HIGHER – ADDITIONAL QUESTION BANK
You have chosen to study: UNIT 3 : Wave Function Please choose a question to attempt from the following: 1 2 3 Back to Unit 3 Menu EXIT

281 WAVE FUNCTION: Question 1
Express cos(x°) + 7sin (x°) in the form kcos(x° - a°) where k>0 and 0  a  90. Hence solve cos(x°) + 7sin (x°) = for 0  x  90. Reveal answer only Go to full solution Go to Marker’s Comments Go to Wave Function Menu Go to Main Menu EXIT

282 WAVE FUNCTION: Question 1
Express cos(x°) + 7sin (x°) in the form kcos(x° - a°) where k>0 and 0  a  90. Hence solve cos(x°) + 7sin (x°) = for 0  x  90. Hence cos(x°) + 7sin (x°) = 52cos(x° °) Reveal answer only (a) Go to full solution Solution is {36.9} Go to Marker’s Comments (b) Go to Wave Function Menu Go to Main Menu EXIT

283 Question 1 Let cos(x°) + 7sin (x°) = kcos(x° - a°)
Express cos(x°) + 7sin (x°) in the form kcos(x° - a°) where k>0 and 0  a  90. Hence solve cos(x°) + 7sin (x°) = 5 for 0  x  90. = k(cosx°cosa° + sinx°sina° ) = (kcosa°)cosx° + (ksina°)sinx° Comparing coefficients kcosa° = 1 & ksina° = 7 So (kcosa°)2 + (ksina°)2 = or k2cos2a° + k2sin2a° = or k2(cos2a° + sin2a°) = 50 or k2 = (cos2a° + sin2a°) = 1 Begin Solution so k = 50 = 252 = 52 Continue Solution Markers Comments Wave Function Menu Back to Home

284 Question 1 so k = 50 = 252 = 52 Express cos(x°) + 7sin (x°)
in the form kcos(x° - a°) where k>0 and 0  a  90. Hence solve cos(x°) + 7sin (x°) = 5 for 0  x  90. ksina° kcosa° = 7 1   so a in Q1! ksina° > 0 so a in Q1 or Q2 kcosa° > 0 so a in Q1 or Q4 tana° = 7 a° = tan-1(7) = 81.9° Hence cos(x°) + 7sin (x°) = 52cos(x° °) Begin Solution Continue Solution Markers Comments Wave Function Menu Back to Home

285 Question 1 Hence cos(x°) + 7sin (x°) = 52cos(x° - 81.9°)
Express cos(x°) + 7sin (x°) in the form kcos(x° - a°) where k>0 and 0  a  90. Hence solve cos(x°) + 7sin (x°) = 5 for 0  x  90. (b) If cos(x°) + 7sin (x°) = 5 then 52cos(x° °) = 5 or cos(x° °) = 1/2 Q1 or Q4 & cos-1(1/2) = 45° Q1: angle = 45° so x° ° = 45° so x° = °  not in range. Begin Solution Continue Solution Q4: angle = 360° - 45° so x° ° = 315° so x° = ° (**) Markers Comments Wave Function Menu Back to Home

286 Question 1 Q4: angle = 360° - 45° so x° - 81.9° = 315°
Express cos(x°) + 7sin (x°) in the form kcos(x° - a°) where k>0 and 0  a  90. Hence solve cos(x°) + 7sin (x°) = 5 for 0  x  90. (**) function repeats every 360° & ° - 360° = 36.9° Solution is {36.9} Begin Solution Continue Solution Markers Comments Wave Function Menu Back to Home

287 Use the formula sheet for the correct expansion: kcos(x° - a°)
Markers Comments Use the formula sheet for the correct expansion: kcos(x° - a°) = kcosx°cosa° + ksinx°sina° (Take care not to omit the “k” term) Let cos(x°) + 7sin (x°) = kcos(x° - a°) = k(cosx°cosa° + sinx°sina° ) = (kcosa°)cosx° + (ksina°)sinx° Comparing coefficients kcosa° = 1 & ksina° = 7 So (kcosa°)2 + (ksina°)2 = or k2cos2a° + k2sin2a° = or k2(cos2a° + sin2a°) = 50 or k2 = (cos2a° + sin2a°) = 1 so k = 50 = 252 = 52 Next Comment Wave Func Menu Back to Home

288 When equating coefficients “square” and “ring”
Markers Comments When equating coefficients “square” and “ring” corresponding coefficients: 1cosx° + 7sinx° = kcosx°cosa° + ksinx°sina° Let cos(x°) + 7sin (x°) = kcos(x° - a°) = k(cosx°cosa° + sinx°sina° ) = (kcosa°)cosx° + (ksina°)sinx° Comparing coefficients kcosa° = 1 & ksina° = 7 So (kcosa°)2 + (ksina°)2 = or k2cos2a° + k2sin2a° = or k2(cos2a° + sin2a°) = 50 or k2 = (cos2a° + sin2a°) = 1 so k = 50 = 252 = 52 Next Comment Wave Func Menu Back to Home

289 State the resulting equations explicitly: kcosa° = 1 ksina° = 7
Markers Comments State the resulting equations explicitly: kcosa° = 1 ksina° = 7 Let cos(x°) + 7sin (x°) = kcos(x° - a°) = k(cosx°cosa° + sinx°sina° ) = (kcosa°)cosx° + (ksina°)sinx° Comparing coefficients kcosa° = 1 & ksina° = 7 So (kcosa°)2 + (ksina°)2 = or k2cos2a° + k2sin2a° = or k2(cos2a° + sin2a°) = 50 or k2 = (cos2a° + sin2a°) = 1 so k = 50 = 252 = 52 Next Comment Wave Func Menu Back to Home

290 k can be found directly:
Markers Comments k can be found directly: Let cos(x°) + 7sin (x°) = kcos(x° - a°) = k(cosx°cosa° + sinx°sina° ) = (kcosa°)cosx° + (ksina°)sinx° Note: k is always positive Comparing coefficients kcosa° = 1 & ksina° = 7 So (kcosa°)2 + (ksina°)2 = or k2cos2a° + k2sin2a° = or k2(cos2a° + sin2a°) = 50 or k2 = (cos2a° + sin2a°) = 1 so k = 50 = 252 = 52 Next Comment Wave Func Menu Back to Home

291 WAVE FUNCTION: Question 2
Express 4sin(x°)–2cos(x°) in the form ksin(x° + a°) where k>0 and 0 < a < 360. Reveal answer only Go to full solution Go to Marker’s Comments Go to Wave Function Menu Go to Main Menu EXIT

292 WAVE FUNCTION: Question 2
Express 4sin(x°)–2cos(x°) in the form ksin(x° + a°) where k>0 and 0 < a < 360. Hence 4sin(x°) – 2cos(x°) = 25sin(x° °) Reveal answer only Go to full solution Go to Marker’s Comments Go to Wave Function Menu Go to Main Menu EXIT

293 Question 2 Let 4sin(x°)–2cos(x°) = ksin(x° + a°)
Express 4sin(x°)–2cos(x°) in the form ksin(x° + a°) where k>0 and 0 < a < 360. = k(sinx°cosa° + cosx°sina° ) = (kcosa°)sinx° + (ksina°)cosx° Comparing coefficients kcosa° = 4 & ksina° = -2 So (kcosa°)2 + (ksina°)2 = (-2)2 or k2cos2a° + k2sin2a° = or k2(cos2a° + sin2a°) = 20 Begin Solution or k2 = (cos2a° + sin2a°) = 1 Continue Solution so k = 20 = 45 = 25 Markers Comments Wave Function Menu Back to Home

294 Question 2 so k = 20 = 45 = 25 Express 4sin(x°)–2cos(x°)
in the form ksin(x° + a°) where k>0 and 0 < a < 360.   so a in Q4! ksina° kcosa° = -2 4 tana° = (-1/2) so Q2 or Q4 ksina° < 0 so a in Q3 or Q4 kcosa° > 0 so a in Q1 or Q4 tan-1(1/2) = 26.6° Q4: a° = 360° – 26.6° = 333.4° Begin Solution Hence 4sin(x°) – 2cos(x°) = 25sin(x° °) Continue Solution Markers Comments Wave Function Menu Back to Home

295 Use the formula sheet for the correct expansion: ksin(x° + a°)
Markers Comments Use the formula sheet for the correct expansion: ksin(x° + a°) = ksinx°cosa° + kcosx°sina° (Take care not to omit the “k” term) Let 4sin(x°)–2cos(x°) = ksin(x° + a°) = k(sinx°cosa° + cosx°sina° ) = (kcosa°)sinx° + (ksina°)cosx° Comparing coefficients kcosa° = 4 & ksina° = -2 So (kcosa°)2 + (ksina°)2 = (-2)2 or k2cos2a° + k2sin2a° = or k2(cos2a° + sin2a°) = 20 or k2 = (cos2a° + sin2a°) = 1 Next Comment so k = 20 = 45 = 25 Wave Func Menu Back to Home

296 When equating coefficients “square” and “ring”
Markers Comments When equating coefficients “square” and “ring” corresponding coefficients: 4sinx° - 2cosx° = ksinx°cosa° + kcosx°sina° Let 4sin(x°)–2cos(x°) = ksin(x° + a°) = k(sinx°cosa° + cosx°sina° ) = (kcosa°)sinx° + (ksina°)cosx° Comparing coefficients kcosa° = 4 & ksina° = -2 State the resulting equations explicitly: kcosa° = ksina° = -2 So (kcosa°)2 + (ksina°)2 = (-2)2 or k2cos2a° + k2sin2a° = or k2(cos2a° + sin2a°) = 20 or k2 = (cos2a° + sin2a°) = 1 Next Comment so k = 20 = 45 = 25 Wave Func Menu Back to Home

297 k can be found directly:
Markers Comments k can be found directly: Let 4sin(x°)–2cos(x°) = ksin(x° + a°) = k(sinx°cosa° + cosx°sina° ) = (kcosa°)sinx° + (ksina°)cosx° Note: k is always positive Comparing coefficients kcosa° = 4 & ksina° = -2 So (kcosa°)2 + (ksina°)2 = (-2)2 or k2cos2a° + k2sin2a° = or k2(cos2a° + sin2a°) = 20 or k2 = (cos2a° + sin2a°) = 1 Next Comment so k = 20 = 45 = 25 Wave Func Menu Back to Home

298 Use the sign of the equations to determine the correct quadrant:
Markers Comments Use the sign of the equations to determine the correct quadrant: kcosa° = 4 (cos +ve) ksina° = -2 (sin -ve) so k = 20 = 45 = 25   so a in Q4! ksina° kcosa° = -2 4 tana° = (-1/2) so Q2 or Q4  cos +ve &sin -ve ksina° < 0 so a in Q3 or Q4 kcosa° > 0 so a in Q1 or Q4 tan-1(1/2) = 26.6° Q4: a° = 360° – 26.6° = 333.4° Hence 4sin(x°) – 2cos(x°) = 25sin(x° °) Next Comment Wave Func Menu Back to Home

299 WAVE FUNCTION: Question 3
The graph below is that of y = 5sin(x°) - 12cos(x°) P y = 5sin(x°) - 12cos(x°) Reveal answer only Go to full solution Go to Marker’s Comms Go to Wave Funct Menu Go to Main Menu Express y = 5sin(x°) - 12cos(x°) in the form y = ksin(x° + a°) where k > 0 and 0  a < 360 Hence find the coordinates of the maximum turning point at P. EXIT

300 WAVE FUNCTION: Question 3
The graph below is that of y = 5sin(x°) - 12cos(x°) P y = 5sin(x°) - 12cos(x°) Reveal answer only Go to full solution Go to Marker’s Comms Go to Wave Funct Menu Go to Main Menu Express y = 5sin(x°) - 12cos(x°) in the form y = ksin(x° + a°) where k > 0 and 0  a < 360 Hence find the coordinates of the maximum turning point at P. Hence 5sin(x°) – 12cos(x°) = 13sin(x° °) EXIT Max TP is at (157.4,13)

301 Question 3 (a) Let 5sin(x°) – 12cos(x°) = ksin(x° + a°)
Express y = 5sin(x°) - 12cos(x°) in the form y = ksin(x° + a°) where k > 0 and 0  a < 360 Hence find the coordinates of the maximum turning point at P. = k(sinx°cosa° + cosx°sina° ) = (kcosa°)sinx° + (ksina°)cosx° Comparing coefficients kcosa° = 5 & ksina° = -12 So (kcosa°)2 + (ksina°)2 = (-12)2 or k2cos2a° + k2sin2a° = or k2(cos2a° + sin2a°) = 169 Begin Solution or k2 = (cos2a° + sin2a°) = 1 Continue Solution so k = 13 Markers Comments Wave Function Menu Back to Home

302 Question 3    so a in Q4! so k = 13
Express y = 5sin(x°) - 12cos(x°) in the form y = ksin(x° + a°) where k > 0 and 0  a < 360 Hence find the coordinates of the maximum turning point at P. ksina° kcosa° = -12 5 tana° = (-12/5) so Q2 or Q4 ksina° < 0 so a in Q3 or Q4 kcosa° > 0 so a in Q1 or Q4 tan-1(12/5) = 67.4° Q4: a° = 360° – 67.4° = 292.6° Hence 5sin(x°) – 12cos(x°) = 13sin(x° °) Begin Solution Continue Solution Markers Comments Wave Function Menu Back to Home

303 Question 3 13sin(x° + 292.6°) is 13. Hence
5sin(x°) – 12cos(x°) = 13sin(x° °) Express y = 5sin(x°) - 12cos(x°) in the form y = ksin(x° + a°) where k > 0 and 0  a < 360 Hence find the coordinates of the maximum turning point at P. The maximum value of 13sin(x° °) is 13. Maximum on a sin graph occurs when angle = 90°. ie x = 90 or x = (**) This is not in the desired range but the function repeats every 360°. Begin Solution Continue Solution Taking = Markers Comments Max TP is at (157.4,13) Wave Function Menu Back to Home

304 Use the formula sheet for the correct expansion: ksin(x° + a°)
Markers Comments Use the formula sheet for the correct expansion: ksin(x° + a°) = ksinx°cosa° + kcosx°sina° (Take care not to omit the “k” term) (a) Let 5sin(x°) – 12cos(x°) = ksin(x° + a°) = k(sinx°cosa° + cosx°sina° ) = (kcosa°)sinx° + (ksina°)cosx° Comparing coefficients kcosa° = 5 & ksina° = -12 So (kcosa°)2 + (ksina°)2 = (-12)2 or k2cos2a° + k2sin2a° = or k2(cos2a° + sin2a°) = 169 or k2 = (cos2a° + sin2a°) = 1 Next Comment so k = 13 Wave Func Menu Back to Home

305 When equating coefficients “square” and “ring”
Markers Comments When equating coefficients “square” and “ring” corresponding coefficients: 5sinx° - 12cosx° = ksinx°cosa° + kcosx°sina° (a) Let 5sin(x°) – 12cos(x°) = ksin(x° + a°) = k(sinx°cosa° + cosx°sina° ) = (kcosa°)sinx° + (ksina°)cosx° Comparing coefficients kcosa° = 5 & ksina° = -12 So (kcosa°)2 + (ksina°)2 = (-12)2 State the resulting equations explicitly: kcosa° = ksina° = -12 or k2cos2a° + k2sin2a° = or k2(cos2a° + sin2a°) = 169 or k2 = (cos2a° + sin2a°) = 1 Next Comment so k = 13 Wave Func Menu Back to Home

306 k can be found directly:
Markers Comments k can be found directly: (a) Let 5sin(x°) – 12cos(x°) = ksin(x° + a°) = k(sinx°cosa° + cosx°sina° ) = (kcosa°)sinx° + (ksina°)cosx° Comparing coefficients kcosa° = 5 & ksina° = -12 Note: k is always positive So (kcosa°)2 + (ksina°)2 = (-12)2 or k2cos2a° + k2sin2a° = or k2(cos2a° + sin2a°) = 169 or k2 = (cos2a° + sin2a°) = 1 Next Comment so k = 13 Wave Func Menu Back to Home

307 Use the sign of the equations to determine the correct quadrant:
Markers Comments Use the sign of the equations to determine the correct quadrant: kcosa° = (cos +ve) ksina° = -12 (sin -ve)   so a in Q4! so k = 13 ksina° kcosa° = -12 5 tana° = (-12/5) so Q2 or Q4 ksina° < 0 so a in Q3 or Q4 kcosa° > 0 so a in Q1 or Q4  cos +ve &sin -ve tan-1(12/5) = 67.4° Q4: a° = 360° – 67.4° = 292.6° Hence 5sin(x°) – 12cos(x°) = 13sin(x° °) Next Comment Wave Func Menu Back to Home

308 be found by considering rules for related functions:
Markers Comments The maximum value of 13sin(x°+292.6°) can also be found by considering rules for related functions: 13sin(x° °) slides 13sinx° graph 292.6° to the left. Maximum value of sinx° occurs at 90° The maximum value of 13sin(x° °) is 13. Maximum on a sin graph occurs when angle = 90°. ie x = 90 or x = (**) This is not in the desired range but the function repeats every 360°. Taking = Max TP is at (157.4,13) Next Comment Wave Func Menu Back to Home

309 Add 360° to bring into domain x = 157.4° 13sin(x° + 292.6°) is 13.
Markers Comments Maximum value of sin(x°+292.6°) occurs at (90° °) i.e at x = ° Add 360° to bring into domain x = 157.4° The maximum value of 13sin(x° °) is 13. Maximum on a sin graph occurs when angle = 90°. ie x = 90 or x = (**) This is not in the desired range but the function repeats every 360°. Taking = Max TP is at (157.4,13) Next Comment Wave Func Menu Back to Home

310 HIGHER – ADDITIONAL QUESTION BANK
You have chosen to study: UNIT 3 : Logs & Exponentials Please choose a question to attempt from the following: 1 2 3 4 5 Back to Unit 3 Menu EXIT

311 LOGS & EXPONENTIALS: Question 1
As a radioactive substance decays the amount of radioactive material remaining after t hours, At , is given by the formula At = A0e-0.161t where A0 is the original amount of material. If 400mg of material remain after 10 hours then determine how much material there was at the start. The half life of the substance is the time required for exactly half of the initial amount to decay. Find the half life to the nearest minute. Reveal answer only Go to full solution Go to Marker’s Comments Go to Logs & Exp Menu EXIT Go to Main Menu

312 LOGS & EXPONENTIALS: Question 1
As a radioactive substance decays the amount of radioactive material remaining after t hours, At , is given by the formula At = A0e-0.161t where A0 is the original amount of material. If 400mg of material remain after 10 hours then determine how much material there was at the start. The half life of the substance is the time required for exactly half of the initial amount to decay. Find the half life to the nearest minute. Initial amount of material = 2000mg Reveal answer only t = 4hrs 18mins Go to full solution Go to Marker’s Comments Go to Logs & Exp Menu EXIT Go to Main Menu

313 Question 1 (a) When t = 10, A10 = 400 so At = A0e-0.161t
becomes A10 = A0e-0.161X10 If 400mg of material remain after 10 hours then determine how much material there was at the start. or = A0e-1.61 and A0 =  e-1.61 ie A0 = … or to 3 sfs Initial amount of material = 2000mg Begin Solution Continue Solution Markers Comments Logs & Exp Menu Back to Home

314 Question 1 (b) For half life At = 1/2A0 so At = A0e-0.161t
becomes 1/2A0 = A0e-0.161t (b) The half life of the substance is the time required for exactly half of the initial amount to decay. Find the half life to the nearest minute. or e-0.161t = 0.5 or ln(e-0.161t ) = ln0.5 ie t = ln0.5 so t = ln0.5  (-0.161) ie t = …hrs ( X 60 = ) Begin Solution Continue Solution or t = 4hrs 18mins Markers Comments Logs & Exp Menu Back to Home

315 The ex is found on the calculator: ex 2nd ln Markers Comments
(a) When t = 10, A10 = 400 so At = A0e-0.161t The ex is found on the calculator: becomes A10 = A0e-0.161X10 2nd ln ex or = A0e-1.61 and A0 =  e-1.61 ie A0 = … or to 3 sfs Initial amount of material = 2000mg Next Comment Logs & Exp Menu Back to Home

316 The half life can be found using any real value:
Markers Comments (b) (b) For half life At = 1/2A0 The half life can be found using any real value: e.g. A0 = At = 1000 so At = A0e-0.161t becomes 1/2A0 = A0e-0.161t or e-0.161t = 0.5 This results in the equation or ln(e-0.161t ) = ln0.5 At = A0e-0.161t 1000 = 2000e-0.161t etc. ie t = ln0.5 so t = ln0.5  (-0.161) ie t = …hrs ( X 60 = ) Next Comment or t = 4hrs 18mins Logs & Exp Menu Back to Home

317 To solve an exponential equation must use logs.
Markers Comments (b) (b) For half life At = 1/2A0 To solve an exponential equation must use logs. A trial and error solution will only be given minimum credit: e.g. 12 = e2x take logs. to both sides ln12 = ln (e2x) log and exponential are inverse functions ln 12 = 2x etc. so At = A0e-0.161t becomes 1/2A0 = A0e-0.161t or e-0.161t = 0.5 or ln(e-0.161t ) = ln0.5 ie t = ln0.5 so t = ln0.5  (-0.161) ie t = …hrs ( X 60 = ) Next Comment or t = 4hrs 18mins Logs & Exp Menu Back to Home

318 LOGS & EXPONENTIALS: Question 2
log10y log10x 1 0.3 The graph illustrates the law y = k xn The line passes through (0,0.3) and (1,0). Find the values of k and n. Reveal answer only Go to full solution Go to Marker’s Comments Go to Logs & Exp Menu EXIT Go to Main Menu

319 LOGS & EXPONENTIALS: Question 2
log10y log10x 1 0.3 The graph illustrates the law y = k xn The line passes through (0,0.3) and (1,0). Find the values of k and n. Reveal answer only Hence k = 2 and n = 0.3 Go to full solution Go to Marker’s Comments Go to Logs & Exp Menu EXIT Go to Main Menu

320 Question 2 log10y log10x Intercept = (0,0.3)
The graph illustrates the law y = k gradient = 0 – 0.3 1 - 0 = -0.3 xn Straight line so in form Y = mX + c with Y = log10y and X = log10x The line passes through (0,0.3) and (1,0). Find the values of k and n. log10y log10x 1 0.3 This becomes log10y = -0.3log10x + 0.3 Or log10y = -0.3log10x + log or log10y = -0.3log10x + log102 Begin Solution or log10y = log10x log102 law3 Continue Solution or log10y = log102x-0.3 Markers Comments law1 Logs & Exp Menu Back to Home

321 Question 2 log10y log10x or log10y = log102x-0.3
law1 The graph illustrates the law y = k so y = 2x-0.3 = 2 x0.3 xn Hence k = 2 and n = 0.3 The line passes through (0,0.3) and (1,0). Find the values of k and n. log10y log10x 1 0.3 Begin Solution Continue Solution Markers Comments Logs & Exp Menu Back to Home

322 It is also possible to find the values of k and n by applying
Markers Comments It is also possible to find the values of k and n by applying the laws of logs to the given equation and substituting two coordinates from the graph: e.g. y = kxn Take logs to both sides log y = log kxn Apply Law 1: log AB = log A + logB Intercept = (0,0.3) gradient = 0 – 0.3 1 - 0 Straight line so in form Y = mX + c with Y = log10y and X = log10x This becomes log10y = -0.3log10x + 0.3 Or log10y = -0.3log10x + log or log10y = -0.3log10x + log102 or log10y = log10x log102 Next Comment or log10y = log102x-0.3 Logs & Exp Menu Back to Home

323 log y = logk + logxn Apply Law 3: logxn = nlogx log y = logk + nlogx
Markers Comments log y = logk + logxn Apply Law 3: logxn = nlogx log y = logk + nlogx log y = nlogx + logk Intercept = (0,0.3) gradient = 0 – 0.3 1 - 0 Straight line so in form Y = mX + c with Y = log10y and X = log10x This becomes log10y = -0.3log10x + 0.3 Or log10y = -0.3log10x + log or log10y = -0.3log10x + log102 or log10y = log10x log102 Next Comment or log10y = log102x-0.3 Logs & Exp Menu Back to Home

324 Two coordinates from the graph:
Markers Comments Two coordinates from the graph: or log10y = log102x-0.3 log10y log10x 1 0.3 so y = 2x-0.3 = 2 x0.3 Hence k = 2 and n = 0.3 Next Comment Straight Line Menu Back to Home

325 Two coordinates from the graph:
Markers Comments Two coordinates from the graph: or log10y = log102x-0.3 log y = nlogx + logk (0,0.3) = n.0 + logk - 1 (1,0) = n.1 + logk so y = 2x-0.3 = 2 x0.3 Hence k = 2 and n = 0.3 Hence solve 1 and 2 to find k and n logk = hence k = 2, and n = - logk hence n= -0.3. Next Comment Straight Line Menu Back to Home

326 LOGS & EXPONENTIALS: Question 3
Solve the equation log3(5) – log3(2x + 1) = -2 Reveal answer only Go to full solution Go to Marker’s Comments Go to Logs & Exp Menu EXIT Go to Main Menu

327 LOGS & EXPONENTIALS: Question 3
Solve the equation log3(5) – log3(2x + 1) = -2 x = 22 Reveal answer only Go to full solution Go to Marker’s Comments Go to Logs & Exp Menu EXIT Go to Main Menu

328 ( ) Question 3 If log3(5) – log3(2x + 1) = -2 Solve the equation
then log 2x + 1 ( ) = -2 law2 so 5 (2x + 1) = 3-2 or 5 (2x + 1) = 1 9 Cross mult we get x = 45 or x = 44 Begin Solution ie x = 22 Continue Solution Markers Comments Logs & Exp Menu Back to Home

329 ( ) To solve an equation involving logs apply the laws so that the
Markers Comments To solve an equation involving logs apply the laws so that the equation is reduced to log=log and the log can be removed and the equation solved: log35 - log3(2x+1) = -2 If log3(5) – log3(2x + 1) = -2 then log 2x + 1 ( ) = -2 so 5 (2x + 1) = 3-2 or 5 (2x + 1) = 1 9 Cross mult Law 2: log = logA - logB A B we get x = 45 Must know: log33 = 1 or x = 44 Next Comment ie x = 22 Logs & Exp Menu Back to Home

330 ( ) 5 log3 = -2 log33 2x+1 Law 3: log = nlogx xn 5 log3 = log33-2 2x+1
Markers Comments log = -2 log33 5 2x+1 If log3(5) – log3(2x + 1) = -2 Law 3: log = nlogx xn then log 2x + 1 ( ) = -2 log = log33-2 5 2x+1 so 5 (2x + 1) = 3-2 The log terms can now be dropped from both sides of the equation and the equation solved: or 5 (2x + 1) = 1 9 Cross mult we get x = 45 or x = 44 Next Comment ie x = 22 Logs & Exp Menu Back to Home

331 ( ) Drop log3 terms = 3-2 5 2x+1 = 5 2x+1 1 9 etc. Markers Comments
If log3(5) – log3(2x + 1) = -2 = 3-2 5 2x+1 then log 2x + 1 ( ) = -2 = 5 2x+1 1 9 so 5 (2x + 1) = 3-2 etc. or 5 (2x + 1) = 1 9 Cross mult we get x = 45 or x = 44 Next Comment ie x = 22 Logs & Exp Menu Back to Home

332 LOGS & EXPONENTIALS: Question 4
The pressure in a leaky tyre drops according to the formula Pt = P0e-kt where P0 is the initial tyre pressure and Pt is the pressure after t hours. If the tyre is inflated to 35psi but this drops to 31 psi in 30 minutes then find the value of k to 3 decimal places. By how many more psi will the pressure drop in the next 15 mins? sssss Reveal answer only Go to full solution Go to Marker’s Comments Go to Logs & Exp Menu EXIT Go to Main Menu

333 LOGS & EXPONENTIALS: Question 4
The pressure in a leaky tyre drops according to the formula Pt = P0e-kt where P0 is the initial tyre pressure and Pt is the pressure after t hours. If the tyre is inflated to 35psi but this drops to 31 psi in 30 minutes then find the value of k to 3 decimal places. By how many more psi will the pressure drop in the next 15 mins? sssss Reveal answer only (a) k = to 3 dps Go to full solution 1.8psi (b) Go to Marker’s Comments Go to Logs & Exp Menu EXIT Go to Main Menu

334 Question 4 We have P0 = 35, Pt = 31 and t = 0.5 (30mins = ½ hr)
If the tyre is inflated to 35psi but this drops to 31 psi in 30 minutes then find the value of k to 3 decimal places. So Pt = P0e-kt becomes 31 = 35e(-0.5k) this becomes e(-0.5k) = 31/35 or lne(-0.5k) = ln(31/35) or k = ln(31/35) or k = ln(31/35)  (-0.5) Begin Solution ie k = to 3 dps Continue Solution Markers Comments Logs & Exp Menu Back to Home

335 Question 4 ie k = 0.243 to 3 dps We now have k = 0.243,
P0 = 31 and t = (15mins = 1/4hr) (b) By how many more psi will the pressure drop in the next 15 mins? using Pt = P0e-0.243t we get P0.25 = 31e(-0.243X0.25) so P0.25 = 29.2 Pressure drop in next 15 mins is 31 – or 1.8psi Begin Solution Continue Solution Markers Comments Logs & Exp Menu Back to Home

336 (a) The ex is found on the calculator: ex 2nd ln Markers Comments
We have P0 = 35, Pt = 31 and t = 0.5 (30mins = ½ hr) 2nd ln ex So Pt = P0e-kt becomes 31 = 35e(-0.5k) this becomes e(-0.5k) = 31/35 or lne(-0.5k) = ln(31/35) or k = ln(31/35) or k = ln(31/35)  (-0.5) ie k = to 3 dps Next Comment Logs & Exp Menu Back to Home

337 To solve an exponential equation must use logs.
Markers Comments To solve an exponential equation must use logs. A trial and error solution will only be given minimum credit: e.g. e(-0.5k) = 31/35 take logs. to both sides ln e(-0.5k) = ln 31/35 log and exponential are inverse functions -0.5k = ln31/35 etc. We have P0 = 35, Pt = 31 and t = 0.5 (30mins = ½ hr) So Pt = P0e-kt becomes 31 = 35e(-0.5k) this becomes e(-0.5k) = 31/35 or lne(-0.5k) = ln(31/35) or k = ln(31/35) or k = ln(31/35)  (-0.5) ie k = to 3 dps Next Comment Logs & Exp Menu Back to Home

338 LOGS & EXPONENTIALS: Question 5
Given that sin = au and cos = av show that u – v = logatan. Reveal answer only Go to full solution Go to Marker’s Comments Go to Logs & Exp Menu EXIT Go to Main Menu

339 LOGS & EXPONENTIALS: Question 5
Given that sin = au and cos = av show that u – v = logatan. u – v = logatan Reveal answer only Go to full solution Go to Marker’s Comments Go to Logs & Exp Menu EXIT Go to Main Menu

340 Question 5 Since sin = au and cos = av Given that sin = au
and cos = av show that u – v = logatan. then u = logasin and v = logacos so u – v = logasin - logacos or u – v = loga(sin/cos) logx – logy = log(x/y) hence u – v = logatan Begin Solution Continue Solution Markers Comments Logs & Exp Menu Back to Home

341 Should be known from standard grade
Markers Comments There are different routes to the same result but all involve correct application of formulas and laws of logs: Since sin = au and cos = av Should be known from standard grade then u = logasin and v = logacos so u – v = logasin - logacos sin  cos  or u – v = loga(sin/cos) e.g. tan = logx – logy = log(x/y) tan = au av hence u – v = logatan Next Comment Logs & Exp Menu Back to Home

342 Take logs to base a to both sides
Markers Comments Take logs to base a to both sides Since sin = au and cos = av loga tan = loga au av then u = logasin and v = logacos Law2: log =logA-logB A B so u – v = logasin - logacos loga tan = loga au - loga av or u – v = loga(sin/cos) Law 3: log = nlogx xn logx – logy = log(x/y) loga tan = uloga a - vloga a hence u – v = logatan logaa=1 loga tan = u - v Next Comment Logs & Exp Menu Back to Home

343 HIGHER – ADDITIONAL QUESTION BANK
You have chosen to study: UNIT 3 : Vectors Please choose a question to attempt from the following: 1 2 3 4 5 Back to Unit 3 Menu EXIT

344 VECTORS: Question 1 P, Q & R are the points (3,0,-5), (-1,1,-2) & (-13,4,7) respectively. Prove that these points are collinear. (b) Given that PS = 7 PQ then find the coordinates of S. Reveal answer only Go to full solution Go to Marker’s Comments Go to Vectors Menu EXIT Go to Main Menu

345 VECTORS: Question 1 P, Q & R are the points (3,0,-5), (-1,1,-2) & (-13,4,7) respectively. Prove that these points are collinear. (b) Given that PS = 7 PQ then find the coordinates of S. (a) PQ and PR are Since multiples of the same Reveal answer only vector and have P as a common point then Go to full solution it follows that P, Q & R are collinear. Go to Marker’s Comments Go to Vectors Menu (b) S = (-25, 7, 16) Go to Main Menu EXIT

346 [ ] [ ] [ ] [ ] [ ] - - Question 1 PQ = q - p =  (a)
[ ] -1 1 -2 3 - PQ = q - p = (a) P, Q & R are the points (3,0,-5), (-1,1,-2) & (-13,4,7) respectively. Prove that these points are collinear. (b) Given that [ ] -4 1 3 = [ ] -13 4 7 3 -2 - PR = r - p = PS = 7 PQ then find [ ] -16 4 9 [ ] -4 1 3 = 4 = the coordinates of S. PQ and PR are Since multiples of the same Begin Solution vector and have P as a common point then Continue Solution it follows that P, Q & R are collinear. Markers Comments Vectors Menu Back to Home

347 [ ] [ ] [ ] Question 1 = 7 = (b) PS = 7  PQ
[ ] -4 1 3 = 7 [ ] -28 7 21 = (b) PS = 7 PQ P, Q & R are the points (3,0,-5), (-1,1,-2) & (-13,4,7) respectively. Prove that these points are collinear. (b) Given that S is (3,0,-5) + [ ] -28 7 21 = (3-28, 0+7, -5+21) = (-25, 7, 16) PS = 7 PQ then find the coordinates of S. Begin Solution Continue Solution Markers Comments Vectors Menu Back to Home

348 [ ] [ ] [ ] [ ] [ ] - - a) Must know result:
Markers Comments a) [ ] -1 1 -2 3 - PQ = q - p = (a) Must know result: Given coordinates of A and B [ ] -4 1 3 = [ ] -13 4 7 3 -2 - PR = r - p = [ ] -16 4 9 [ ] -4 1 3 = 4 = PQ and PR are Since multiples of the same Next Comment vector and have P as a common point then Vectors Menu it follows that P, Q & R are collinear. Back to Home

349 [ ] [ ] [ ] [ ] [ ] - - Must be able to state explicitly
Markers Comments Must be able to state explicitly the result for collinearity: [ ] -1 1 -2 3 - PQ = q - p = (a) [ ] -4 1 3 = [ ] -13 4 7 3 -2 - PR = r - p = [ ] -16 4 9 [ ] -4 1 3 = 4 = PQ and PR are Since multiples of the same Next Comment vector and have P as a common point then Vectors Menu it follows that P, Q & R are collinear. Back to Home

350 [ ] [ ] [ ] b) An alternative approach is to
Markers Comments b) An alternative approach is to form a vector equation and solve it: [ ] -4 1 3 = 7 [ ] -28 7 21 = (b) PS = 7 PQ S is (3,0,-5) + [ ] -28 7 21 = (3-28, 0+7, -5+21) = (-25, 7, 16) Next Comment Vectors Menu Back to Home

351 [ ] [ ] [ ] b) Markers Comments = 7 = (b) PS = 7  PQ S is (3,0,-5) +
[ ] -4 1 3 = 7 [ ] -28 7 21 = (b) PS = 7 PQ S is (3,0,-5) + [ ] -28 7 21 = (3-28, 0+7, -5+21) = (-25, 7, 16) Next Comment Vectors Menu Back to Home

352 VECTORS: Question 2 E, F & G are the points (4,3,-1), (5,-1,2) & (8,-1,-1) respectively. Find FE and FG in component form. (b) Hence, or otherwise, find the size of angle EFG. Reveal answer only Go to full solution Go to Marker’s Comments Go to Vectors Menu EXIT Go to Main Menu

353 [ ] [ ] VECTORS: Question 2
E, F & G are the points (4,3,-1), (5,-1,2) & (8,-1,-1) respectively. Find FE and FG in component form. (b) Hence, or otherwise, find the size of angle EFG. FE (a) [ ] -1 4 -3 = [ ] 3 -3 FG Reveal answer only = Go to full solution so  = ° Go to Marker’s Comments Go to Vectors Menu EXIT Go to Main Menu

354 [ ] [ ] [ ] [ ] - - Question 2 FE = e - f =  (a)
[ ] 4 3 -1 5 2 - FE = e - f = (a) E, F & G are the points (4,3,-1), (5,-1,2) & (8,-1,-1) respectively. Find [ ] -1 4 -3 = FE and FG in component form. [ ] 8 -1 5 2 - FG = g - f = Hence, or otherwise, find the size of angle EFG. [ ] 3 -3 = Begin Solution Continue Solution Markers Comments Vectors Menu Back to Home

355 [ ] Question 2 . (b) Let angle EFG =  E, F & G are the points
(4,3,-1), (5,-1,2) & (8,-1,-1) respectively. Find E F G ie FE and FG in component form. [ ] -1 4 -3 3 . FE . FG = Hence, or otherwise, find the size of angle EFG. = (-1 X 3) + (4 X 0) + (-3 X (-3)) = = 6 Begin Solution |FE| = ((-1) (-3)2) = 26 Continue Solution |FG| = ( (-3)2) Markers Comments = 18 Vectors Menu Back to Home

356 Question 2 Given that FE.FG = |FE||FG|cos  E, F & G are the points
(4,3,-1), (5,-1,2) & (8,-1,-1) respectively. Find then cos = FE.FG |FE||FG| = 6 26 18 FE and FG in so  = cos-1(6  26  18) = ° component form. Hence, or otherwise, find the size of angle EFG. Begin Solution Continue Solution Markers Comments Vectors Menu Back to Home

357 [ ] Ensure vectors are calculated from the vertex: E F G .
Markers Comments Ensure vectors are calculated from the vertex: (b) Let angle EFG =  E F G ie F G E [ ] -1 4 -3 3 . FE . FG = = (-1 X 3) + (4 X 0) + (-3 X (-3)) = = 6 |FE| = ((-1) (-3)2) Next Comment = 26 |FG| = ( (-3)2) Vectors Menu = 18 Back to Home

358 Refer to formula sheet and relate formula to given variables:
Markers Comments Refer to formula sheet and relate formula to given variables: Given that FE.FG = |FE||FG|cos then cos = FE.FG |FE||FG| so  = cos-1(6  26  18) = ° Next Comment Vectors Menu Back to Home

359 [ ], [ ]and [ ]resp. VECTORS: Question 3 PQRSTUVW is a cuboid in which
PQ , PS & PW are represented by the vectors    [ ], 4 2 [ ]and -2 [ ]resp. 9 A is 1/3 of the way up ST & B is the midpoint of UV. ie SA:AT = 1:2 & VB:BU = 1:1. Find the components of PA & PB and hence the size of angle APB. Reveal answer only Go to full solution Go to Marker’s Comments Go to Vectors Menu EXIT

360 [ ], [ ]and [ ]resp. VECTORS: Question 3 PQRSTUVW is a cuboid in which
PQ , PS & PW are represented by the vectors    [ ], 4 2 [ ]and -2 [ ]resp. 9 A is 1/3 of the way up ST & B is the midpoint of UV. ie SA:AT = 1:2 & VB:BU = 1:1. Find the components of PA & PB and hence the size of angle APB. Reveal answer only |PA| = 29 Go to full solution |PB| = 106 Go to Marker’s Comments = ° APB Go to Vectors Menu EXIT

361 [ ], [ ]and [ ]resp. [ ] [ ] = [ ] [ ] [ ]+ [ ]= [ ] + + Question 3
PA = PS + SA = PS + 1/3ST PQRSTUVW is a cuboid in which PQ , PS & PW are represented by vectors    [ ], 4 2 [ ]and -2 [ ]resp. 9 PS + 1/3PW = [ ] -2 4 [ ] = 3 + [ ] -2 4 3 = A is 1/3 of the way up ST & B is the midpoint of UV. ie SA:AT = 1:2 & VB:BU = 1:1. PB = PQ + QV + VB Find the components of PA & PB and hence the size of angle APB. PQ + PW + 1/2PS = [ ] 4 2 [ ]+ 9 + [ ]= -1 [ ] 3 4 9 Begin Solution = Continue Solution Markers Comments Vectors Menu Back to Home

362 [ ] [ ] [ ], [ ]and [ ]resp. [ ] Question 3 . PA =  PB = 
[ ] -2 4 3 [ ] 3 4 9 PB = PQRSTUVW is a cuboid in which PQ , PS & PW are represented by vectors    [ ], 4 2 [ ]and -2 [ ]resp. 9 (b) Let angle APB =  A P B ie A is 1/3 of the way up ST & B is the midpoint of UV. ie SA:AT = 1:2 & VB:BU = 1:1. Find the components of PA & PB and hence the size of angle APB. PA . PB = [ ] -2 4 3 9 . Begin Solution = (-2 X 3) + (4 X 4) + (3 X 9) Continue Solution = Markers Comments = 37 Vectors Menu Back to Home

363 [ ], [ ]and [ ]resp. Question 3 PA .  PB = 37
PQRSTUVW is a cuboid in which PQ , PS & PW are represented by vectors    [ ], 4 2 [ ]and -2 [ ]resp. 9 |PA| = ((-2) ) = 29 |PB| = ( ) = 106 Given that PA.PB = |PA||PB|cos A is 1/3 of the way up ST & B is the midpoint of UV. ie SA:AT = 1:2 & VB:BU = 1:1. then cos = PA.PB |PA||PB| = 37 29 106 Find the components of PA & PB and hence the size of angle APB. so  = cos-1(37  29  106) Begin Solution Continue Solution = ° Markers Comments Vectors Menu Back to Home

364 [ ] [ ] [ ] Ensure vectors are calculated from the vertex: B P A .
Markers Comments Ensure vectors are calculated from the vertex: PA = [ ] -2 4 3 [ ] 3 4 9 PB = P A B (b) Let angle APB =  A P B ie PA . PB = [ ] -2 4 3 9 . Next Comment = (-2 X 3) + (4 X 4) + (3 X 9) = Vectors Menu = 37 Back to Home

365 Refer to formula sheet and relate formula to given variables:
Markers Comments Refer to formula sheet and relate formula to given variables: PA . PB = |PA| = ((-2) ) = 29 |PB| = ( ) = 106 Given that PA.PB = |PA||PB|cos then cos = PA.PB |PA||PB| so  = cos-1(37  29  106) Next Comment Vectors Menu = ° Back to Home

366 VECTORS: Question 4 b c a An equilateral triangle has sides 4 units long which are represented by the vectors a , b & c as shown. Find b.(a + c) & comment on your answer (ii) b.(a – c) Reveal answer only Go to full solution Go to Marker’s Comments Go to Vectors Menu EXIT Go to Main Menu

367 VECTORS: Question 4 b c a An equilateral triangle has sides 4 units long which are represented by the vectors a , b & c as shown. Find b.(a + c) & comment on your answer (ii) b.(a – c) Dot product = 0 so b & (a + c) are perpendicular. (ii) b.(a – c) = b.b = 42 = 16 Reveal answer only Go to full solution Go to Marker’s Comments Go to Vectors Menu EXIT Go to Main Menu

368 Question 4 NB: each angle is 60° but vectors should be “tail to tail”
An equilateral triangle has sides 4 units long which are represented by the vectors a , b & c . b c 60° 120° Find b.(a + c) & comment b.(a – c) (i) b.(a + c) = b.a + b.c = |b||a|cos1 + |b||c|cos2 = 4X4Xcos60° + 4X4Xcos120° = 16 X ½ X (-½ ) Begin Solution = 8 + (-8) Continue Solution = 0 Markers Comments Dot product = 0 so b & (a + c) are perpendicular. Vectors Menu Back to Home

369 Question 4 NB: a – c = a + (-c) or b . An equilateral triangle has
sides 4 units long which are represented by the vectors a , b & c . (ii) b.(a – c) = b.b = |b||b|cos0 = |b|2 Find b.(a + c) & comment b.(a – c) = 42 = 16 Begin Solution Continue Solution Markers Comments Vectors Menu Back to Home

370 This geometric question is based on the scalar product definition
Markers Comments This geometric question is based on the scalar product definition and the distributive law: NB: each angle is 60° but vectors should be “tail to tail” b c 60° 120° and a(b+c) = ab + ac (i) b.(a + c) = b.a + b.c No other results should be applied. = |b||a|cos1 + |b||c|cos2 = 4X4Xcos60° + 4X4Xcos120° = 16 X ½ X (-½ ) = 8 + (-8) = 0 Next Comment Dot product = 0 so b & (a + c) are perpendicular. Vectors Menu Back to Home

371 Ensure all scalar products are calculated from the vertex:
Markers Comments Ensure all scalar products are calculated from the vertex: NB: each angle is 60° but vectors should be “tail to tail” b c 60° 120° (i) b.(a + c) = b.a + b.c = |b||a|cos1 + |b||c|cos2 = 4X4Xcos60° + 4X4Xcos120° = 16 X ½ X (-½ ) = 8 + (-8) = 0 Next Comment Dot product = 0 so b & (a + c) are perpendicular. Vectors Menu Back to Home

372 VECTORS: Question 5 a = 2i + j – 3k, b = -i + 10k & c = -2i + j + k.
(a) Find (i) 2a + b in component form (ii) | 2a + b | (b) Show that 2a + b and c are perpendicular. Reveal answer only Go to full solution Go to Marker’s Comments Go to Vectors Menu EXIT Go to Main Menu

373 [ ] VECTORS: Question 5 = 29
a = 2i + j – 3k, b = -i + 10k & c = -2i + j + k. (a) Find (i) 2a + b in component form (ii) | 2a + b | (b) Show that 2a + b and c are perpendicular. [ ] 3 2 4 = (a)(i) 2a + b Reveal answer only Go to full solution (ii) | 2a + b | = 29 Go to Marker’s Comments (b) Go to Vectors Menu Since the dot product is zero it follows that the vectors are perpendicular. Go to Main Menu EXIT

374 [ ] [ ] [ ] Question 5 + (a)(i) 2a + b = a = 2i + j – 3k, b = -i + 10k
[ ] 2 1 -3 -1 10 + (a)(i) 2a + b = a = 2i + j – 3k, b = -i + 10k & c = -2i + j + k. [ ] 4 2 -6 -1 10 + = (a) Find 2a + b in component form | 2a + b | [ ] 3 2 4 = Show that 2a + b and c are perpendicular. Begin Solution Continue Solution Markers Comments Vectors Menu Back to Home

375 Question 5 = 29 (ii) | 2a + b | = (32 + 22 + 42)
a = 2i + j – 3k, b = -i + 10k & c = -2i + j + k. = 29 (a) Find 2a + b in component form | 2a + b | Show that 2a + b and c are perpendicular. Begin Solution Continue Solution Markers Comments Vectors Menu Back to Home

376 [ ] [ ] . Question 5 (b) (2a + b ).c = a = 2i + j – 3k, b = -i + 10k
4 [ ] -2 1 . (b) (2a + b ).c = a = 2i + j – 3k, b = -i + 10k & c = -2i + j + k. = (3 X (-2))+(2 X 1)+(4 X 1) (a) Find 2a + b in component form | 2a + b | = = 0 Since the dot product is zero it follows that the vectors are perpendicular. Show that 2a + b and c are perpendicular. Begin Solution Continue Solution Markers Comments Vectors Menu Back to Home

377 [ ] [ ] [ ] When a vector is given in i , j , k form change to
Markers Comments When a vector is given in i , j , k form change to component form: e.g. 5i - 2j + k = [ ] 2 1 -3 -1 10 + (a)(i) 2a + b = [ ] 4 2 -6 -1 10 + = [ ] 3 2 4 = Next Comment Vectors Menu Back to Home

378 Must know formula for the magnitude of a vector:
Markers Comments Must know formula for the magnitude of a vector: (ii) | 2a + b | = ( ) = 29 Next Comment Vectors Menu Back to Home

379 [ ] [ ] . Must know condition for perpendicular vectors:
Markers Comments Must know condition for perpendicular vectors: [ ] 3 2 4 [ ] -2 1 . (b) (2a + b ).c = = (3 X (-2))+(2 X 1)+(4 X 1) = = 0 Since the dot product is zero it follows that the vectors are perpendicular. Next Comment Vectors Menu Back to Home

380 HIGHER – ADDITIONAL QUESTION BANK
You have chosen to study: UNIT 3 : Further Calculus Please choose a question to attempt from the following: 1 2 3 4 Back to Unit 3 Menu EXIT

381 FURTHER CALCULUS : Question 1
Given that y = 3sin(2x) – 1/2cos(4x) then find dy/dx . Reveal answer only Go to full solution Go to Marker’s Comments Go to Further Calculus Menu Go to Main Menu EXIT

382 FURTHER CALCULUS : Question 1
Given that y = 3sin(2x) – 1/2cos(4x) then find dy/dx . = 6cos(2x) + 2sin(4x) Reveal answer only Go to full solution Go to Marker’s Comments Go to Further Calculus Menu Go to Main Menu EXIT

383 Question 1 Given that y = 3sin(2x) – 1/2cos(4x) then find dy/dx .
OUTER / INNER Differentiate outer then inner Given that y = 3sin(2x) – 1/2cos(4x) then find dy/dx . y = 3sin(2x) – 1/2cos(4x) dy/dx = 3cos(2x) X (-1/2sin(4x)) X 4 = 6cos(2x) + 2sin(4x) Begin Solution Continue Solution Markers Comments Further Calc Menu Back to Home

384 Check formula sheet for correct result:
Markers Comments Check formula sheet for correct result: 3sin(2x) 1/2cos(4x) OUTER / INNER Differentiate outer then inner y sin(ax) acos(ax) cos(ax) asin(ax) y = 3sin(2x) – 1/2cos(4x) Relate formula to given variables dy/dx = 3cos(2x) X (-1/2sin(4x)) X 4 = 6cos(2x) + 2sin(4x) Next Comment Further Calc Menu Back to Home

385 When applying the “chain rule” “Peel an onion”
Markers Comments When applying the “chain rule” “Peel an onion” 3sin(2x) 1/2cos(4x) OUTER / INNER Differentiate outer then inner 3sin(2x) 1/2cos(4x) OUTER / INNER Differentiate outer then inner y = 3sin(2x) – 1/2cos(4x) dy/dx = 3cos(2x) X (-1/2sin(4x)) X 4 = 6cos(2x) + 2sin(4x) e.g. 3sin cos 2x Next Comment Further Calc Menu Back to Home

386 FURTHER CALCULUS : Question 2
Given that g(x) = (6x – 5) then evaluate g´(9) . Reveal answer only Go to full solution Go to Marker’s Comments Go to Further Calculus Menu Go to Main Menu EXIT

387 FURTHER CALCULUS : Question 2
Given that g(x) = (6x – 5) then evaluate g´(9) . = 3/7 Reveal answer only Go to full solution Go to Marker’s Comments Go to Further Calculus Menu Go to Main Menu EXIT

388 Question 2 Given that g(x) = (6x – 5) then evaluate g´(9) .
g(x) = (6x – 5) = (6x – 5)1/2 Given that g(x) = (6x – 5) then evaluate g´(9) . (6x – 5)1/2 outer / inner diff outer then inner g´(x) = 1/2(6x – 5)-1/2 X 6 = 3(6x – 5)-1/2 = (6x – 5) g´(9) = (6X9 – 5) Begin Solution Continue Solution = 49 Markers Comments Further Calc Menu = 3/7 Back to Home

389 Apply the laws of indices to replace the sign
Markers Comments Apply the laws of indices to replace the sign g(x) = (6x – 5) = (6x – 5)1/2 (6x – 5)1/2 outer / inner diff outer then inner Apply the chain rule Learn the rule for differentiation Multiply by the power then reduce the power by 1 g´(x) = 1/2(6x – 5)-1/2 X 6 = 3(6x – 5)-1/2 = (6x – 5) g´(9) = (6X9 – 5) Next Comment = 49 Further Calc Menu = 3/7 Back to Home

390 Apply the laws of indices to return power to a positive value
Markers Comments Apply the laws of indices to return power to a positive value then the root g(x) = (6x – 5) = (6x – 5)1/2 (6x – 5)1/2 outer / inner diff outer then inner g´(x) = 1/2(6x – 5)-1/2 X 6 Will usually work out to an exact value without need to use calculator. = 3(6x – 5)-1/2 = (6x – 5) g´(9) = (6X9 – 5) Next Comment = 49 Further Calc Menu = 3/7 Back to Home

391 FURTHER CALCULUS : Question 3
A curve for which dy/dx = -12sin(3x) passes through the point (/3,-2). Express y in terms of x. Reveal answer only Go to full solution Go to Marker’s Comments Go to Further Calculus Menu Go to Main Menu EXIT

392 FURTHER CALCULUS : Question 3
A curve for which dy/dx = -12sin(3x) passes through the point (/3,-2). Express y in terms of x. Reveal answer only So y = 4cos(3x) + 2 Go to full solution Go to Marker’s Comments Go to Further Calculus Menu Go to Main Menu EXIT

393 through the point (/3,-2). Express y in terms of x.
Question 3 if dy/dx = -12sin(3x) A curve for which dy/dx = -12sin(3x) passes through the point (/3,-2). Express y in terms of x. then y =  -12sin(3x) dx = 1/3 X 12cos(3x) + C = 4cos(3x) + C At the point (/3,-2) y = 4cos(3x) + C becomes = 4cos(3 X /3) + C or = 4cos + C or = C ie C = 2 Begin Solution Continue Solution So y = 4cos(3x) + 2 Markers Comments Further Calc Menu Back to Home

394 Learn the result that integration undoes differentiation: i.e. given
Markers Comments Learn the result that integration undoes differentiation: i.e. given if dy/dx = -12sin(3x) then y =  -12sin(3x) dx = 1/3 X 12cos(3x) + C = 4cos(3x) + C At the point (/3,-2) y = 4cos(3x) + C becomes = 4cos(3 X /3) + C or = 4cos + C or = C ie C = 2 Next Comment So y = 4cos(3x) + 2 Further Calc Menu Back to Home

395 Check formula sheet for correct result: if dy/dx = -12sin(3x)
Markers Comments Check formula sheet for correct result: if dy/dx = -12sin(3x) then y =  -12sin(3x) dx sinax cos(ax) a = 1/3 X 12cos(3x) + C + c = 4cos(3x) + C cos(ax) sin(ax) a + c At the point (/3,-2) y = 4cos(3x) + C becomes = 4cos(3 X /3) + C or = 4cos + C Do not forget the constant of integration. or = C ie C = 2 Next Comment So y = 4cos(3x) + 2 Further Calc Menu Back to Home

396  FURTHER CALCULUS : Question 4
Find the derivative of y = (2x3 + 1)2/3 where x > 0. Hence find x2 (2x3 + 1)1/3 dx Reveal answer only Go to full solution Go to Marker’s Comments Go to Further Calculus Menu Go to Main Menu EXIT

397  FURTHER CALCULUS : Question 4 = 4x2
Find the derivative of y = (2x3 + 1)2/3 where x > 0. Hence find x2 (2x3 + 1)1/3 dx = /4(2x3 + 1)2/ C Reveal answer only Go to full solution Go to Marker’s Comments Go to Further Calculus Menu Go to Main Menu EXIT

398 Question 4 (a) if y = (2x3 + 1)2/3 (2x3 + 1)2/3 outer inner
diff outer then inner Find the derivative of y = (2x3 + 1)2/3 where x > 0. then dy/dx = 2/3 (2x3 + 1)-1/3 X 6x2 = x2 (2x3 + 1)1/3 Begin Solution Continue Solution Markers Comments Further Calc Menu Back to Home

399     Question 4 = ¼ X (b) From (a) it follows that (b) Hence find
dx = (2x3 + 1)2/ C x2 (2x3 + 1)1/3 dx now x2 (2x3 + 1)1/3 dx 4x2 (2x3 + 1)1/3 dx = ¼ X Begin Solution = /4(2x3 + 1)2/ C Continue Solution Markers Comments Further Calc Menu Back to Home

400 Markers Comments . Apply the chain rule “Peel an onion”
(a) if y = (2x3 + 1)2/3 (2x3 + 1)2/3 outer inner diff outer then inner (2x3 + 1)2/3 outer inner diff outer then inner then dy/dx = 2/3 (2x3 + 1)-1/3 X 6x2 = x2 (2x3 + 1)1/3 e.g. Next Comment Further Calc Menu Back to Home

401    Learn the result that integration undoes differentiation:
Markers Comments b) Learn the result that integration undoes differentiation: i.e. given (b) From (a) it follows that 4x2 (2x3 + 1)1/3 dx = (2x3 + 1)2/ C now x2 (2x3 + 1)1/3 dx 4x2 (2x3 + 1)1/3 dx = ¼ X = /4(2x3 + 1)2/ C Next Comment Further Calc Menu Back to Home


Download ppt "HIGHER – ADDITIONAL QUESTION BANK"

Similar presentations


Ads by Google